Endocrine System Practice NCLEX Questions & study set

अब Quizwiz के साथ अपने होमवर्क और परीक्षाओं को एस करें!

A patient is prescribed levothyroxine. To promote optimal absorption, the nurse should instruct the patient to take the medication at which time? 1 0600 2 1200 3 1600 4 2100

1 0600 For maximum absorption, levothyroxine should be taken first thing in the morning on an empty stomach 30 minutes before breakfast. 1200, 1600, and 2100 may not result in adequate absorption.

A client with diabetes mellitus who refuses to take insulin as prescribed exhibits markedly increased blood glucose levels after a meal. The nurse caring for the client anticipates that which initial body response to elevated glucose levels will worsen the situation for the client? 1. Glycogenolysis 2. Gluconeogenesis 3. Binding of glucose onto cell membranes 4. Transport of glucose across cell membranes

1. Glycogenolysis

A patient has just begun long-term corticosteroid therapy. The nurse determines that the patient requires further education when making which statement? 1 "I may need to monitor my blood sugar more frequently." 2 "If I begin to gain weight I should stop taking my medication." 3 "It is important that I stay away from people who have contagious illnesses." 4 "I understand my appearance may change as fat tissue increases in my face and trunk."

2 "If I begin to gain weight I should stop taking my medication." Corticosteroids should be gradually tapered and not stopped suddenly to avoid life-threatening adrenal insufficiency. Corticosteroids may lead to insulin resistance and increased gluconeogenesis by the liver, and therefore the patient may need to monitor for blood sugar increase. Corticosteroids decrease the inflammatory response and delay healing, and therefore the patient is more susceptible to infections. Adipose tissue accumulates in the trunk, face, and cervical spine as a result of corticosteroid therapy.

A nurse is caring for a client with a dysfunctional thyroid gland and is concerned that the client will exhibit signs of thyroid storm. Which is an early indicator of this complication? 1. Constipation 2. Bradycardia 3. Hyperreflexia 4. Low-grade temperature

3. Hyperreflexia

Bronze-colored skin with hyperpigmentation in sun-exposed areas along with the other clinical findings indicates _________ disease.

Addison's The drug prescribed in this case should be fludrocortisone

A nurse is instructing a client with DM I about exercise. What is an appropriate statement by the nurse?

Always where a medical identification tag while exercising

The nurse is preparing to teach a 43-year-old man who is newly diagnosed with type 2 diabetes about home management of the disease. Which action should the nurse take first? Ask the patient's family to participate in the diabetes education program. Assess the patient's perception of what it means to have diabetes mellitus. Demonstrate how to check glucose using capillary blood glucose monitoring. Discuss the need for the patient to actively participate in diabetes management.

B - Before planning teaching, the nurse should assess the patient's interest in and ability to self-manage the diabetes. After assessing the patient, the other nursing actions may be appropriate, but planning needs to be individualized to each patient.

5. A nurse is providing teaching to a client who is scheduled for a phentolamine blocking test. This test supports a diagnosis for which of the following disorders? A. Addison's disease B. Diabetes mellitus C. Cushing's disease D. Pheochromocytoma

D. Pheochromocytoma

The nurse suspects that a client with DM may be non-compliant with the treatment plan. The nurse should know that a reliable test to evaluate if the client is routinely compliant with the prescribed regimen is what?

Glucosylated hemoglobin levels or Hemoglobin A1C

acute tubular necrosis (ATN)

a medical condition involving the death of renal tubules caused by renal ischemia, nephrotoxic injury, and sepsis; the most common intrarenal cause of acute kidney injury (AKI) (Lewis page 1071)

CKD mineral and bone disorder (CKD-MBD)

a systemic disorder of mineral and bone metabolism caused by progressive deterioration in kidney function (Lewis page 1078)

anuria

no urine output (Lewis page 1071)

continuous renal replacement therapy (CRRT)

provides a means by which solutes and fluids can be removed slowly and continuously in the hemodynamically unstable patient; usually used to treat acute renal failure (Lewis page 1091)

A nurse is caring for a client with DM II. What findings indicate to the nurse of hyperglycemia?

sweating and increased urination

end-stage kidney (renal) disease (ESRD)

the last stage of kidney failure that occurs when the GFR is <15 mL/min (Lewis page 1076)

uremia

the presence of excessive amounts of urea and other nitrogenous waste products in the blood; renal function declines to the point that symptoms develop in multiple body systems (Lewis page 1076)

The nurse teaches the patient receiving levothyroxine that symptoms of drug toxicity include which of the following? Select all that apply. 1 Chest pain 2 Weight gain 3 Nervousness 4 Tachycardia 5 Cold intolerance 6 Mental sluggishness

1 Chest pain 3 Nervousness 4 Tachycardia Levothyroxine is a synthetic thyroid hormone used to treat hypothyroidism. The signs of overtreatment, or levothyroxine toxicity, are the same as the signs of hyperthyroidism, a state of increased metabolism and increased tissue sensitivity to sympathetic nervous system stimulation. Signs of overtreatment of hypothyroidism with levothyroxine include chest pain, nervousness, and tachycardia. Weight gain, cold intolerance, and mental sluggishness are signs of hypothyroidism.

A client with diabetes mellitus who takes insulin is seen in the health care clinic. The client tells the clinic nurse that after the insulin injection, the insulin seems to leak through the skin. The nurse would appropriately determine the problem by asking the client which question? 1. "Are you rotating the injection site?" 2. "Are you aspirating before you inject the insulin?" 3. "Are you using a 1-inch needle to give the injection?" 4. "Are you placing an air bubble in the syringe before injection?"

1. "Are you rotating the injection site?" Rationale: The client should be instructed that insulin injection sites should be rotated within 1 anatomical area before moving on to another area. This rotation process promotes uniform absorption of insulin and reduces the chances of irritation. The remaining options are not associated with the condition (skin leakage of insulin) presented in the question.

The home health nurse visits a client with a diagnosis of type 1 diabetes mellitus. The client relates a history of vomiting and diarrhea and tells the nurse that no food has been consumed for the last 24 hours. Which additional statement by the client indicates a need for further teaching? 1. "I need to stop my insulin." 2. "I need to increase my fluid intake." 3. "I need to monitor my blood glucose every 3 to 4 hours." 4. "I need to call the health care provider (HCP) because of these symptoms."

1. "I need to stop my insulin." Rationale: When a client with diabetes mellitus is unable to eat normally because of illness, the client still should take the prescribed insulin or oral medication. The client should consume additional fluids and should notify the HCP. The client should monitor the blood glucose level every 3 to 4 hours. The client should also monitor the urine for ketones during illness.

The nurse is teaching a client with hyperparathyroidism how to manage the condition at home. Which response by the client indicates the need for additional teaching? 1. "I should limit my fluids to 1 liter per day." 2. "I should use my treadmill or go for walks daily." 3. "I should follow a moderate-calcium, high-fiber diet." 4. "My alendronate helps to keep calcium from coming out of my bones."

1. "I should limit my fluids to 1 liter per day." Rationale: In hyperparathyroidism, clients experience excess parathyroid hormone (PTH) secretion. A role of PTH in the body is to maintain serum calcium homeostasis. When PTH levels are high, there is excess bone resorption (calcium is pulled from the bones). In clients with elevated serum calcium levels, there is a risk of nephrolithiasis. One to 2 liters of fluids daily should be encouraged to protect the kidneys and decrease the risk of nephrolithiasis. Moderate physical activity, particularly weight-bearing activity, minimizes bone resorption and helps to protect against pathological fracture. Walking, as an exercise, should be encouraged in the client with hyperparathyroidism. Clients should follow a moderate-calcium, high-fiber diet. Even though serum calcium is already high, clients should follow a moderate-calcium diet because a low-calcium diet will surge PTH. Calcium causes constipation, so a diet high in fiber is recommended. Alendronate is a bisphosphate that inhibits bone resorption. In bone resorption, bone is broken down and calcium is deposited into the serum.

A client received 5 units of aspart insulin (NovoLog) subcutaneously just before eating lunch at 12:00 pm. The nurse should assess the client for a hypoglycemic reaction at which times? 1. Between 1:00 and 3:00 pm 2. 10 minutes after administration 3. Between 4:00 pm and 12:00 am 4. Between 8:00 pm and 10:00 pm

1. Between 1:00 and 3:00 pm

The nurse is performing an assessment on a client with a diagnosis of myxedema (hypothyroidism). Which assessment finding should the nurse expect to note in this client? 1. Dry skin 2. Thin, silky hair 3. Bulging eyeballs 4. Fine muscle tremors

1. Dry skin Rationale: Myxedema is a deficiency of thyroid hormone. The client will present with a puffy, edematous face, especially around the eyes (periorbital edema), along with coarse facial features; dry skin; and dry, coarse hair and eyebrows. The remaining options are noted in the client with hyperthyroidism.

The nurse is providing dietary instructions to help with diabetes control for a client newly diagnosed with diabetes mellitus who will be taking insulin. The nurse should provide the client with which best instruction? 1. Eat meals at approximately the same time each day. 2. Adjust meal times depending on blood glucose levels. 3. Vary meal times if insulin is not administered at the same time every day. 4. Avoid being concerned about the time of meals so long as snacks are taken on time.

1. Eat meals at approximately the same time each day. Rationale: Mealtimes must be approximately the same each day to maintain a stable blood glucose level. The client should not be instructed that mealtimes can be varied depending on blood glucose levels, insulin administration, or consumption of snacks.

A client has been diagnosed with hyperthyroidism. The nurse monitors for which signs and symptoms indicating a complication of this disorder? Select all that apply. 1. Fever 2. Nausea 3. Lethargy 4. Tremors 5. Confusion 6. Bradycardia

1. Fever 2. Nausea 4. Tremors 5. Confusion Rationale: Thyroid storm is an acute and life-threatening complication that occurs in a client with uncontrollable hyperthyroidism. Signs and symptoms of thyroid storm include elevated temperature (fever), nausea, and tremors. In addition, as the condition progresses, the client becomes confused. The client is restless and anxious and experiences tachycardia.

The nurse is caring for a postoperative parathyroidectomy client. Which client complaint would indicate that a life-threatening complication may be developing, requiring notification of the health care provider immediately? 1. Laryngeal stridor 2. Abdominal cramps 3. Difficulty in voiding 4. Mild to moderate incisional pain

1. Laryngeal stridor/laryngeal nerve damage

The nurse should include which interventions in the plan of care for a client with hypothyroidism? Select all that apply. 1. Provide a warm environment for the client. 2. Instruct the client to consume a low-fat diet. 3. A thyroid-releasing inhibitor will be prescribed. 4. Encourage the client to consume a well-balanced diet. 5. Instruct the client that thyroid replacement therapy will be needed. 6. Instruct the client that episodes of chest pain are expected to occur.

1. Provide a warm environment for the client. 2. Instruct the client to consume a low-fat diet. 4. Encourage the client to consume a well-balanced diet. 5. Instruct the client that thyroid replacement therapy will be needed.

A client's serum blood glucose level is 48 mg/dL. The nurse would expect to note which as an additional finding when assessing this client? 1. Slurred speech 2. Increased thirst 3. Increased appetite 4. Increased urination

1. Slurred speech

A client with diabetes mellitus is being discharged following treatment for hyperglycemic hyperosmolar state (HHS) precipitated by acute illness. The client tells the nurse, "will call the health care provider (HCP) the next time I can't eat for more than a day or so." Which statement reflects the most appropriate analysis of this client's level of knowledge? 1. The client needs immediate education before discharge. 2. The client requires follow-up teaching regarding the administration of oral antidiabetics. 3. The client's statement is inaccurate, and he or she should be scheduled for outpatient diabetic counseling. 4. The client's statement is inaccurate, and he or she should be scheduled for educational home health visits.

1. The client needs immediate education before discharge. Rationale: If the client becomes ill and cannot retain fluids or food for a period of 4 hours, the HCP should be notified. The client's statement indicates a need for immediate education to prevent hyperosmolar hyperglycemic syndrome (HHS), a life-threatening emergency. Although all of the other options may be true, the most appropriate analysis is that the client requires immediate education.

The nurse is providing home care instructions to the client with a diagnosis of Cushing's syndrome and prepares a list of instructions for the client. Which instructions should be included on the list? Select all that apply. 1. The signs and symptoms of hypoadrenalism 2. The signs and symptoms of hyperadrenalism 3. Instructions to take the medications exactly as prescribed 4. The importance of maintaining regular outpatient follow-up care 5. A reminder to read the labels on over-the-counter medications before purchase

1. The signs and symptoms of hypoadrenalism 2. The signs and symptoms of hyperadrenalism 3. Instructions to take the medications exactly as prescribed 4. The importance of maintaining regular outpatient follow-up care Rationale: The client with Cushing's syndrome should be instructed to take the medications exactly as prescribed. The nurse should emphasize the importance of continuing medications, consulting with the health care provider (HCP) before purchasing any over-the-counter medications, and maintaining regular outpatient follow-up care. The nurse also should instruct the client in the signs and symptoms of both hypoadrenalism and hyperadrenalism.

A client with an endocrine disorder complains of weight loss and diarrhea, and says that he can "feel his heart beating in his chest." The nurse interprets that which gland is most likely responsible for these symptoms? 1. Thyroid 2. Pituitary 3. Parathyroid 4. Adrenal cortex

1. Thyroid

The nurse is monitoring a client who was diagnosed with type 1 diabetes mellitus and is being treated with NPH and regular insulin. Which client complaints would alert the nurse to the presence of a possible hypoglycemic reaction? Select all that apply. 1. Tremors 2. Anorexia 3. Irritability 4. Nervousness 5. Hot, dry skin 6. Muscle cramps

1. Tremors 3. Irritability 4. Nervousness Rationale: Decreased blood glucose levels produce autonomic nervous system symptoms, which are manifested classically as nervousness, irritability, and tremors. Option 5 is more likely to occur with hyperglycemia. Options 2 and 6 are unrelated to the manifestations of hypoglycemia. In hypoglycemia, usually the client feels hunger.

The patient has a prescription for levothyroxine 37.5 mcg. Available are 0.075 mg tablets. How many tablets should the nurse administer? 1 0.25 tablet 2 0.5 tablet 3 0.75 tablet 4 1 tablet

2 0.5 tablet First, convert 0.075 mg to mcg, which equals 75 mcg. Using ratio and proportion, multiply 37.5 by x and multiply 75 × 1 to yield 37.5x = 75. Divide 75 by 37.5 to yield 0.5 tablet.

The nurse is caring for a patient after thyroidectomy. What are the nursing interventions for this patient? Select all that apply. 1 Place the patient in Fowler's position. 2 Monitor vital signs and calcium levels. 3 Check for muscular twitching or tingling in the toes. 4 Assess the patient for hemorrhage every six hours. 5 Assess the patient for irregular breathing or neck swelling.

2 Monitor vital signs and calcium levels. 3 Check for muscular twitching or tingling in the toes. 5 Assess the patient for irregular breathing or neck swelling. The nurse should monitor the patient's vital signs and calcium levels. The patient should be assessed for muscular twitching or tingling in the toes, which are signs of tetany secondary to hypoparathyroidism. The nurse should assess the patient every 2 hours for 24 hours for signs of hemorrhage or tracheal compression such as irregular breathing, neck swelling, frequent swallowing, sensations of fullness at the incision site, choking, and blood on the anterior or posterior dressings. The patient should be placed in semi-Fowler's position and the head should be supported with pillows.

The family of a bedridden client with type 2 diabetes mellitus and a chronic kidney disease calls a nurse to report symptoms of headache, polydipsia, and increased lethargy. Which most important question should the nurse ask the family to determine a possible problem? 1. "What is the client's urine output?" 2. "What is the client's capillary blood glucose level?" 3. "Has there been any change in the dietary intake?" 4. "Have you increased the amount of fluids provided?"

2. "What is the client's capillary blood glucose level?"

The home care nurse is visiting a client newly diagnosed with diabetes mellitus. The client tells the nurse that he is planning to eat a dinner meal at a local restaurant this week. He asks the nurse if eating at a restaurant will affect diabetic control and if this is allowed. Which nursing response is most appropriate? 1. "You are not allowed to eat in restaurants." 2. "You should order a half-portion meal and have fresh fruit for dessert." 3. "If you plan to eat in a restaurant, you need to skip the lunchtime meal." 4. "You should increase your daily dose of insulin by half on the day that you plan to eat in the restaurant."

2. "You should order a half-portion meal and have fresh fruit for dessert." Rationale: Clients with diabetes mellitus are instructed to make adjustments in their total daily intake to plan for meals at restaurants or parties. Some useful strategies include ordering a half-portion, salads with dressing on the side, fresh fruit for dessert, and baked or steamed entrees. Clients are not instructed to skip meals or increase their prescribed insulin dosage.

The nursing instructor asks a nursing student to identify the risk factors associated with the development of thyrotoxicosis. The student demonstrates understanding of the risk factors by identifying an increased risk for thyrotoxicosis in which client? 1. A client with hypothyroidism 2. A client with Graves' disease who is having surgery 3. A client with diabetes mellitus scheduled for a diagnostic test 4. A client with diabetes mellitus scheduled for débridement of a foot ulcer

2. A client with Graves' disease who is having surgery Rationale: Thyrotoxicosis usually is seen in clients with Graves' disease in whom the symptoms are precipitated by a major stressor. This complication typically occurs during periods of severe physiological or psychological stress such as trauma, sepsis, delivery, or major surgery. It also must be recognized as a potential complication after thyroidectomy. The client conditions in the remaining options are not associated with thyrotoxicosis.

The nurse is providing instructions to a client newly diagnosed with diabetes mellitus. The nurse gives the client a list of the signs of hyperglycemia. Which specific sign of this complication should be included on the list? 1. Shakiness 2. Increased thirst 3. Profuse sweating 4. Decreased urine output

2. Increased thirst Rationale: The classic signs of hyperglycemia include polydipsia, polyuria, and polyphagia. Profuse sweating and shakiness would be noted in a hypoglycemic condition.

The nurse is performing an assessment on a client with a diagnosis of hyperthyroidism. Which assessment finding should the nurse expect to note in this client? 1. Dry skin 2. Bulging eyeballs 3. Periorbital edema 4. Coarse facial features

2. Bulging eyeballs Rationale: Hyperthyroidism is clinically manifested by goiter (increase in the size of the thyroid gland) and exophthalmos (bulging eyeballs). Other clinical manifestations include nervousness, fatigue, weight loss, muscle cramps, and heat intolerance. Additional signs found in this disorder include tachycardia; shortness of breath; excessive sweating; fine muscle tremors; thin, silky hair and thin skin; infrequent blinking; and a staring appearance.

A client with a diagnosis of diabetic ketoacidosis (DKA) is being treated in the emergency department. Which findings would the nurse expect to note as confirming this diagnosis? Select all that apply. 1. Increase in pH 2. Comatose state 3. Deep, rapid breathing 4. Decreased urine output 5. Elevated blood glucose level 6. Low plasma bicarbonate level

2. Comatose state 3. Deep, rapid breathing 5. Elevated blood glucose level 6. Low plasma bicarbonate level Rationale: Because of the profound deficiency of insulin associated with DKA, glucose cannot be used for energy and the body breaks down fat as a secondary source of energy. Ketones, which are acid byproducts of fat metabolism, build up and the client experiences a metabolic ketoacidosis. High serum glucose contributes to an osmotic diuresis and the client becomes severely dehydrated. If untreated, the client will become comatose due to severe dehydration, acidosis, and electrolyte imbalance. Kussmaul's respirations, the deep rapid breathing associated with DKA, is a compensatory mechanism by the body. The body attempts to correct the acidotic state by blowing off carbon dioxide (CO2), which is an acid. In the absence of insulin, the client will experience severe hyperglycemia. Option 1 is incorrect because in acidosis the pH would be low. Option 4 is incorrect because a high serum glucose will result in an osmotic diuresis and the client will experience polyuria.

A client with diabetes mellitus demonstrates acute anxiety when first admitted to the hospital for the treatment of hyperglycemia. What is the most appropriate intervention to decrease the client's anxiety? 1. Administer a sedative. 2. Convey empathy, trust, and respect toward the client. 3. Ignore the signs and symptoms of anxiety so that they will soon disappear. 4. Make sure that the client knows all the correct medical terms to understand what is happening.

2. Convey empathy, trust, and respect toward the client. Rationale: Anxiety is a subjective feeling of apprehension, uneasiness, or dread. The appropriate intervention is to address the client's feelings related to the anxiety. Administering a sedative is not the most appropriate intervention and does not address the source of the client's anxiety. The nurse should not ignore the client's anxious feelings. Anxiety needs to be managed before meaningful client education can occur.

A client is admitted to an emergency department, and a diagnosis of myxedema coma is made. Which action would the nurse prepare to carry out initially? 1. Warm the client. 2. Maintain a patent airway. 3. Administer thyroid hormone. 4. Administer fluid replacement.

2. Maintain a patent airway. Rationale: (ABCs of healthcare!) Myxedema coma is a rare but serious disorder that results from persistently low thyroid production. Coma can be precipitated by acute illness, rapid withdrawal of thyroid medication, anesthesia and surgery, hypothermia, and the use of sedatives and opioid analgesics. In myxedema coma, the initial nursing action is to maintain a patent airway. Oxygen should be administered, followed by fluid replacement, keeping the client warm, monitoring vital signs, and administering thyroid hormones by the intravenous route.

A client has overactivity of the thyroid gland. The nurse plans care, knowing that the client will experience which effects from this hormonal excess? 1. Weight gain 2. Nutritional deficiencies 3. Low blood glucose levels 4. Increased body fat stores

2. Nutritional deficiencies

The nurse is caring for a client admitted to the hospital with uncontrolled type 1 diabetes mellitus. In the event that diabetic ketoacidosis (DKA) does occur, the nurse anticipates that which medication would most likely be prescribed? 1. Glucagon 2. Regular insulin 3. Glyburide (DiaBeta) 4. Neutral protamine Hagedorn (NPH) insulin

2. Regular insulin Rationale: Giving regular insulin by the intravenous route is the treatment of choice for DKA. A short-acting insulin is the only insulin that can be given intravenously because it can be titrated to the client's blood glucose levels. Glucagon is used to treat hypoglycemia because it increases blood glucose levels, and glyburide is an oral hypoglycemic agent used to treat type 2 diabetes mellitus; both agents are inappropriate. NPH insulin is an intermediate-acting insulin and therefore is not appropriate for treatment of DKA.

The nurse teaches a client with diabetes mellitus about differentiating between hypoglycemia and ketoacidosis. The client demonstrates an understanding of the teaching by stating that a form of glucose should be taken if which symptoms develop? Select all that apply. 1. Polyuria 2. Shakiness 3. Palpitations 4. Blurred vision 5. Lightheadedness 6. Fruity breath odor

2. Shakiness 3. Palpitations 5. Lightheadedness Rationale: Shakiness, palpitations, and lightheadedness are signs/symptoms of hypoglycemia and would indicate the need for food or glucose. Polyuria, blurred vision, and a fruity breath odor are manifestations of hyperglycemia.

The nurse is caring for a client after hypophysectomy and notes clear nasal drainage from the client's nostril. The nurse should take which initial action? 1. Lower the head of the bed. 2. Test the drainage for glucose. 3. Obtain a culture of the drainage. 4. Continue to observe the drainage.

2. Test the drainage for glucose Rationale: After hypophysectomy, the client should be monitored for rhinorrhea, which could indicate a cerebrospinal fluid leak. If this occurs, the drainage should be collected and tested for the presence of cerebrospinal fluid. Cerebrospinal fluid contains glucose, and if positive, this would indicate that the drainage is cerebrospinal fluid. The head of the bed should remain elevated to prevent increased intracranial pressure. Clear nasal drainage would not indicate the need for a culture. Continuing to observe the drainage without taking action could result in a serious complication.

A newly diagnosed client with diabetes mellitus is started on a two-dose insulin protocol combination of short- and intermediate-acting insulin injected twice daily. What portion of the total dose is given before breakfast and what portion before the evening meal? 1. Half before breakfast and half before the evening meal 2. Two thirds before breakfast and one third before the evening meal 3. One third before breakfast and two thirds before the evening meal 4. Three fourths before breakfast and one fourth before the evening meal

2. Two thirds before breakfast and one third before the evening meal

The nurse is teaching the patient with adrenocortical insufficiency and the caregiver about management of corticosteroid therapy. What should the nurse tell the patient and the caregiver? 1 Assess for cataracts every two years. 2 Decrease the dose of corticosteroids when stressed. 3 Recognize edema and ways to restrict sodium intake. 4 Plan a diet high in concentrated simple carbohydrates.

3 Recognize edema and ways to restrict sodium intake. The nurse should teach the patient to recognize edema and ways to restrict sodium intake to less than 2000 mg/day if edema occurs. The nurse should ask the patient to see an eye specialist yearly to assess for cataracts. The patient should recognize the need for an increased dose of corticosteroids when stressed. The nurse should teach the patient and caregiver to plan a diet high in protein, calcium, and potassium but low in fat and concentrated simple carbohydrates such as sugar, honey, syrups, and candy.

When discussing long-term management of Addison's disease with a patient, the nurse includes which self-care management measures? Select all that apply. 1 The patient will need to follow a low-sodium diet. 2 When taking antacids, the patient may need to decrease corticosteroid medication. 3 The patient must notify the health care provider whenever experiencing vomiting or diarrhea. 4 The patient will need to take extra medication when experiencing either physical or emotional stress. 5 The patient or patient's caregiver will need to administer corticosteroids subcutaneously in the case of an emergency, and the patient cannot take hormone replacements orally.

3 The patient must notify the health care provider whenever experiencing vomiting or diarrhea. 4 The patient will need to take extra medication when experiencing either physical or emotional stress. Vomiting and diarrhea can deplete cortisol levels and parenteral replacement may be needed. The patient with Addison's disease is unable to tolerate physical or emotional stress without exogenous corticosteroids and may need to increase medication at these times. Patients with Addison's disease will need to increase their sodium intake, because they are at risk for hyponatremia. Antacid intake will necessitate increased corticosteroid hormone therapy. Corticosteroids must be given intramuscularly when the patient is unable to take them orally.

The nurse is caring for a client who is scheduled to have a thyroidectomy and provides instructions to the client about the surgical procedure. Which client statement indicates an understanding of the nurse's instructions? 1. "I expect to experience some tingling of my toes, fingers, and lips after surgery." 2. "I will definitely have to continue taking antithyroid medications after this surgery." 3. "I need to place my hands behind my neck when I have to cough or change positions." 4. "I need to turn my head and neck front, back, and laterally every hour for the first 12 hours after surgery."

3. "I need to place my hands behind my neck when I have to cough or change positions."

A home health nurse is visiting a client with type 1 diabetes mellitus. The client tells the nurse that he is not feeling well and has had a "respiratory infection" for the past week, which seems to be getting worse. After interviewing the client, what should be the initial nursing action? 1. Notify the health care provider. 2. Document the assessment data. 3. Check the client's blood glucose. 4. Obtain the client's sputum for culture and sensitivity.

3. Check the client's blood glucose.

A client with type 1 diabetes mellitus calls the nurse to report recurrent episodes of hypoglycemia with exercising. Which statement by the client indicates an inadequate understanding of the peak action of NPH insulin and exercise? 1. "The best time for me to exercise is after I eat." 2. "The best time for me to exercise is after breakfast." 3. "The best time for me to exercise is mid- to late afternoon." 4. "The best time for me to exercise is after my morning snack."

3. "The best time for me to exercise is mid- to late afternoon." Rationale: Clients should avoid exercise during the peak time of insulin. NPH insulin peaks at 4 to 12 hours; therefore, afternoon exercise takes place during the peak of the medication. Exercise is an important part of diabetes management. It promotes weight loss, decreases insulin resistance, and helps to control blood glucose levels. A hypoglycemic reaction may occur in response to increased exercise, so clients should exercise either an hour after mealtime or after consuming a 10- to 15-gram carbohydrate snack, and they should check their blood glucose level before exercising.

The nurse is reviewing the laboratory test results for a client with a diagnosis of Cushing's syndrome. Which laboratory finding would the nurse expect to note in this client? 1. A platelet count of 200,000 mm3 (200 × 109/L) 2. A blood glucose level of 110 mg/dL (6.28 mmol/L) 3. A potassium (K+) level of 3.0 mEq/L (3.0 mmol/L) 4. A white blood cell (WBC) count of 6000 mm3 (6 × 109/L)

3. A potassium (K+) level of 3.0 mEq/L (3.0 mmol/L) Rationale: The client with Cushing's syndrome experiences hypokalemia, hyperglycemia, an elevated WBC count, and elevated plasma cortisol and adrenocorticotropic hormone levels. These abnormalities are caused by the effects of excess glucocorticoids and mineralocorticoids in the body. The laboratory values listed in the remaining options would not be noted in the client with Cushing's syndrome.

The nurse is monitoring a client with diabetes mellitus for signs of hypoglycemia. Which manifestations are associated with this complication? 1. Slow pulse; lethargy; warm, dry skin 2. Elevated pulse; lethargy; warm, dry skin 3. Elevated pulse; shakiness; cool, clammy skin 4. Slow pulse, confusion, increased urine output

3. Elevated pulse; shakiness; cool, clammy skin Rationale: Signs and symptoms of mild hypoglycemia include tachycardia; shakiness; and cool, clammy skin. The remaining options do not specify the manifestations of hypoglycemia.

The nurse is preparing a client with a new diagnosis of hypothyroidism for discharge. The nurse determines that the client understands discharge instructions if the client states that which symptoms are associated with this diagnosis? Select all that apply. 1. Tremors 2. Weight loss 3. Feeling cold 4. Loss of body hair 5. Persistent lethargy 6. Puffiness of the face

3. Feeling cold 4. Loss of body hair 5. Persistent lethargy 6. Puffiness of the face Rationale: Feeling cold, hair loss, lethargy, and facial puffiness are signs of hypothyroidism. Tremors and weight loss are signs of hyperthyroidism.

A hospitalized client is experiencing an episode of hypoglycemia. The nurse plans care, knowing that which is the physiological mechanism that should take place to combat the decline in the blood glucose level? 1. Decreased cortisol release 2. Increased insulin secretion 3. Increased glucagon secretion 4. Decreased epinephrine release

3. Increased glucagon secretion

A client's serum blood glucose level is 389 mg/dL. The nurse would expect to note which as an additional finding when assessing this client? 1. Unsteady gait 2. Slurred speech 3. Increased thirst 4. Cold, clammy skin

3. Increased thirst

The nurse should include which interventions in the plan of care for a client with hypothyroidism? Select all that apply. 1. Provide a cool environment for the client. 2. Instruct the client to consume a high-fat diet. 3. Instruct the client about thyroid replacement therapy. 4. Encourage the client to consume fluids and high-fiber foods in the diet. 5. Inform the client that iodine preparations will be prescribed to treat the disorder. 6. Instruct the client to contact the health care provider (HCP) if episodes of chest pain occur.

3. Instruct the client about thyroid replacement therapy. 4. Encourage the client to consume fluids and high-fiber foods in the diet. 6. Instruct the client to contact the health care provider (HCP) if episodes of chest pain occur. Rationale: The clinical manifestations of hypothyroidism are the result of decreased metabolism from low levels of thyroid hormone. Interventions are aimed at replacement of the hormone and providing measures to support the signs and symptoms related to decreased metabolism. The client often has cold intolerance and requires a warm environment. The nurse encourages the client to consume a well-balanced diet that is low in fat for weight reduction and high in fluids and high-fiber foods to prevent constipation. Iodine preparations may be used to treat hyperthyroidism. Iodine preparations decrease blood flow through the thyroid gland and reduce the production and release of thyroid hormone; they are not used to treat hypothyroidism. The client is instructed to notify the HCP if chest pain occurs because it could be an indication of overreplacement of thyroid hormone.

The nurse is caring for a client after thyroidectomy. The client expresses concern about the postoperative voice hoarseness she is experiencing and asks if the hoarseness will subside. The nurse should provide the client with which information? 1. The hoarseness is permanent. 2. It indicates nerve damage. 3. It is normal during this time and will subside. 4. It will worsen before it subsides, which may take 6 months.

3. It is normal during this time and will subside. Rationale: Hoarseness in the postoperative period usually is the result of laryngeal pressure or edema and will resolve within a few days. The client should be reassured that the effects are transitory. The other options are incorrect.

A client has just been admitted to the nursing unit following thyroidectomy. Which assessment is the priority for this client? 1. Hypoglycemia 2. Level of hoarseness 3. Respiratory distress 4. Edema at the surgical site

3. Respiratory distress Rationale: Thyroidectomy is the removal of the thyroid gland, which is located in the anterior neck. It is very important to monitor airway status, as any swelling to the surgical site could cause respiratory distress. Although all of the options are important for the nurse to monitor, the priority nursing action is to monitor the airway.

A client with type 1 diabetes mellitus is to begin an exercise program, and the nurse is providing instructions regarding the program. Which instruction should the nurse include in the teaching plan? 1. Try to exercise before mealtimes. 2. Administer insulin after exercising. 3. Take a blood glucose test before exercising. 4. Exercise is best performed during peak times of insulin.

3. Take a blood glucose test before exercising.

A client with type 1 diabetes mellitus is to begin an exercise program, and the nurse is providing instructions regarding the program. Which instruction should the nurse include in the teaching plan? 1. Try to exercise before mealtimes. 2. Administer insulin after exercising. 3. Take a blood glucose test before exercising. 4. Exercise is best performed during peak times of insulin.

3. Take a blood glucose test before exercising. Rationale: A blood glucose test performed before exercising provides the client with information regarding the need to consume a snack before exercising. Exercising during the peak times of insulin or before mealtime places the client at risk for hypoglycemia. Insulin should be administered as prescribed.

The nurse is teaching a patient with Addison's disease about corticosteroid therapy. The nurse should prioritize which of these teaching points? 1 "Plan a high-carbohydrate diet." 2 "Increase your daily intake of sodium." 3 "Decrease your daily intake of calcium." 4 "Do not stop taking the medication abruptly."

4 "Do not stop taking the medication abruptly." The patient should be instructed to not stop the medication abruptly because this can cause adverse side effects. Patients taking corticosteroids should not consume a high-carbohydrate diet, because corticosteroids increase blood sugar. Patients should also increase their daily intake of calcium to prevent bone loss due to the side effects of corticosteroids. Patients should also decrease, not increase, their daily intake of sodium to avoid fluid retention.

A patient with a severe pounding headache has been diagnosed with hypertension that is not responding to traditional treatment. What should the nurse expect as the next step in management of this patient? 1 Administration of β-blocker medications 2 Abdominal palpation to search for a tumor 3 Administration of potassium-sparing diuretics 4 A 24-hour urine collection for fractionated metanephrines

4 A 24-hour urine collection for fractionated metanephrines Pheochromocytoma should be suspected when hypertension does not respond to traditional treatment. The 24-hour urine collection for fractionated metanephrines is simple and reliable, with elevated values in 95% of people with pheochromocytoma. In a patient with pheochromocytoma preoperatively an α-adrenergic receptor blocker is used to reduce blood pressure. Abdominal palpation is avoided to avoid a sudden release of catecholamines and severe hypertension. Potassium-sparing diuretics are not needed; most likely they would be used for hyperaldosteronism, which is another cause of hypertension.

A nurse creating a plan of care for a patient with Addison's disease expects that primary treatment will include: 1 Blood transfusions 2 Ablation of the thyroid 3 Oral calcium supplementation 4 Adrenocorticosteroid replacement therapy

4 Adrenocorticosteroid replacement therapy Because Addison's disease results from a deficiency of adrenocorticosteroid hormones, steroid therapy is the primary treatment. Blood transfusions, thyroid ablation, and oral calcium supplements are not primary treatments for Addison's disease.

The nurse is caring for a client scheduled for a transsphenoidal hypophysectomy. The preoperative teaching instructions should include which statement? 1. "Your hair will need to be shaved." 2. "You will receive spinal anesthesia." 3. "You will need to ambulate after surgery." 4. "Brushing your teeth needs to be avoided for at least 2 weeks after surgery."

4. "Brushing your teeth needs to be avoided for at least 2 weeks after surgery." Rationale: A transsphenoidal hypophysectomy is a surgical approach that uses the nasal sinuses and nose for access to the pituitary gland. Based on the location of the surgical procedure, spinal anesthesia would not be used. In addition, the hair would not be shaved. Although ambulating is important, specific to this procedure is avoiding brushing the teeth to prevent disruption of the surgical site. Gentle mouth care every 4 hours is required until teeth can be brushed again.

A client with type 1 diabetes mellitus is having trouble remembering the types, duration, and onset of the action of insulin. The client tells the nurse that the family members have not been supportive. Which response by the nurse is best? 1. "What is it that you don't understand?" 2. "You can't always depend on your family to help." 3. "It's not really necessary for you to remember this." 4. "Let me go over the types of insulin with you again."

4. "Let me go over the types of insulin with you again." Rationale: Reinforcement of knowledge and behaviors is vital to the success of the client's self-care. All of the other options do not address the need for client instructions and are not therapeutic responses.

A client with diabetes mellitus has been instructed in the dietary exchange system. The client asks the nurse if bacon is allowed in the diet. Which nursing response is most appropriate? 1. "Bacon is not allowed." 2. "Bacon is much too high in fat." 3. "Bacon may be eaten if you eliminate one meat item from your diet." 4. "One strip of bacon may be eaten if you eliminate 1 teaspoon of butter."

4. "One strip of bacon may be eaten if you eliminate 1 teaspoon of butter."

The nurse is interviewing a client with type 2 diabetes mellitus. Which statement by the client indicates an understanding of the treatment for this disorder? 1. "I take oral insulin instead of shots." 2. "By taking these medications, I am able to eat more." 3. "When I become ill, I need to increase the number of pills I take." 4. "The medications I'm taking help release the insulin I already make."

4. "The medications I'm taking help release the insulin I already make."

A client is undergoing an oral glucose tolerance test. The nurse interprets that the client's results are compatible with diabetes mellitus if the glucose level is at which value after 120 minutes (2 hours)? 1. 80 mg/dL 2. 110 mg/dL 3. 130 mg/dL 4. 160 mg/dL

4. 160 mg/dL

The nurse is performing an assessment on a client with a diagnosis of Cushing's syndrome. Which should the nurse expect to note on assessment of the client? 1. Skin atrophy 2. The presence of sunken eyes 3. Drooping on 1 side of the face 4. A rounded "moonlike" appearance to the face

4. A rounded "moonlike" appearance to the face Rationale: With excessive secretion of adrenocorticotropic hormone (ACTH) and chronic corticosteroid use, the person with Cushing's syndrome develops a rounded moonlike face; prominent jowls; red cheeks; and hirsutism on the upper lip, lower cheek, and chin. The remaining options are not associated with the assessment findings in Cushing's syndrome.

The nurse is caring for a client admitted to the emergency department with diabetic ketoacidosis (DKA). In the acute phase, the nurse plans for which priority intervention? 1. Correct the acidosis. 2. Administer 5% dextrose intravenously. 3. Apply a monitor for an electrocardiogram. 4. Administer short-duration insulin intravenously.

4. Administer short-duration insulin intravenously. Rationale: Lack of insulin (absolute or relative) is the primary cause of DKA. Treatment consists of insulin administration (short- or rapid-acting), intravenous fluid administration (normal saline initially, not 5% dextrose), and potassium replacement, followed by correcting acidosis. Cardiac monitoring is important due to alterations in potassium levels associated with DKA and its treatment, but applying an electrocardiogram monitor is not the priority action.

The nurse is reviewing the health care provider (HCP) prescriptions for a client with a diagnosis of diabetes mellitus who has been hospitalized for treatment of an infected foot ulcer. The nurse expects to note which finding in the HCP prescriptions? 1. A decreased-calorie diet 2. An increased-calorie diet 3. A decreased amount of NPH daily insulin 4. An increased amount of NPH daily insulin

4. An increased amount of NPH daily insulin Rationale: Infection is a physiological stressor that can cause an increase in the level of epinephrine in the body. An increase in epinephrine causes an increase in blood glucose levels. When the client is under stress, such as when an infection is present, an increase in the dose of insulin will be required to facilitate the transport of excess glucose into the cells. The client will not necessarily need an adjustment in the daily diet.

The nurse is developing a plan of care for a client who is scheduled for a thyroidectomy. The nurse focuses on psychosocial needs, knowing that which is likely to occur in the client? 1. Infertility 2. Gynecomastia 3. Sexual dysfunction 4. Body image changes

4. Body image changes Rationale: Because of the location of the incision in the neck area, many clients are afraid of thyroid surgery for fear of having a visible large scar postoperatively. Having all or part of the thyroid gland removed will not cause the client to experience gynecomastia. Sexual dysfunction and infertility could occur if the entire thyroid is removed and the client is not placed on thyroid replacement medications

The nurse has documented the problem of body image distortion for a client with a diagnosis of Cushing's syndrome. The nurse identifies nursing interventions related to this problem and includes these interventions in the plan of care. Which nursing intervention is inappropriate? 1. Encourage the client's expression of feelings. 2. Assess the client's understanding of the disease process. 3. Encourage family members to share their feelings about the disease process. 4. Encourage the client to recognize that the body changes need to be dealt with.

4. Encourage the client to recognize that the body changes need to be dealt with.

The nurse is caring for a client with a diagnosis of Addison's disease and is monitoring the client for signs of addisonian crisis. The nurse should assess the client for which manifestation that would be associated with this crisis? 1. Agitation 2. Diaphoresis 3. Restlessness 4. Severe abdominal pain

4. Severe abdominal pain Rationale: Addisonian crisis is a serious life-threatening response to acute adrenal insufficiency that most commonly is precipitated by a major stressor. The client in addisonian crisis may demonstrate any of the signs and symptoms of Addison's disease, but the primary problems are sudden profound weakness; severe abdominal, back, and leg pain; hyperpyrexia followed by hypothermia; peripheral vascular collapse; coma; and renal failure. The remaining options do not identify clinical manifestations associated with addisonian crisis.

A client newly diagnosed with diabetes mellitus has been stabilized with daily insulin injections. A nurse prepares a discharge teaching plan regarding the insulin and plans to reinforce which concept? 1. Always keep insulin vials refrigerated. 2. Ketones in the urine signify a need for less insulin. 3. Increase the amount of insulin before excessive exercise. 4. Systematically rotate insulin injections within one anatomical site.

4. Systematically rotate insulin injections within one anatomical site.

The nurse is assessing the learning readiness of a client newly diagnosed with diabetes mellitus. Which behavior indicates to the nurse that the client is not ready to learn? 1. The client asks if the spouse may attend the teaching session. 2. The client asks appropriate questions about what will be taught. 3. The client asks for written materials about diabetes mellitus before class. 4. The client complains of fatigue whenever the nurse plans a teaching session.

4. The client complains of fatigue whenever the nurse plans a teaching session.

In order to assist an older diabetic patient to engage in moderate daily exercise, which action is most important for the nurse to take? A. Determine what type of activities the patient enjoys. B. Remind the patient that exercise will improve self-esteem. C. Teach the patient about the effects of exercise on glucose level. D. Give the patient a list of activities that are moderate in intensity.

A - Because consistency with exercise is important, assessment for the types of exercise that the patient finds enjoyable is the most important action by the nurse in ensuring adherence to an exercise program. The other actions will also be implemented but are not the most important in improving compliance.

Which patient action indicates a good understanding of the nurse's teaching about the use of an insulin pump? A. The patient programs the pump for an insulin bolus after eating. B. The patient changes the location of the insertion site every week. C. The patient takes the pump off at bedtime and starts it again each morning. D. The patient plans for a diet that is less flexible when using the insulin pump.

A - In addition to the basal rate of insulin infusion, the patient will adjust the pump to administer a bolus after each meal, with the dosage depending on the oral intake. The insertion site should be changed every 2 or 3 days. There is more flexibility in diet and exercise when an insulin pump is used. The pump will deliver a basal insulin rate 24 hours a day.

A 32-year-old patient with diabetes is starting on intensive insulin therapy. Which type of insulin will the nurse discuss using for mealtime coverage? A. Lispro (Humalog) B. Glargine (Lantus) C. Detemir (Levemir) D. NPH (Humulin N)

A - Rapid- or short-acting insulin is used for mealtime coverage for patients receiving intensive insulin therapy. NPH, glargine, or detemir will be used as the basal insulin.

Which statement by the patient indicates a need for additional instruction in administering insulin? A. "I need to rotate injection sites among my arms, legs, and abdomen each day." B. "I can buy the 0.5 mL syringes because the line markings will be easier to see." C. "I should draw up the regular insulin first after injecting air into the NPH bottle." D. "I do not need to aspirate the plunger to check for blood before injecting insulin."

A - Rotating sites is no longer recommended because there is more consistent insulin absorption when the same site is used consistently. The other patient statements are accurate and indicate that no additional instruction is needed.

5. A nurse is providing teaching to a client who has a new diagnosis of diabetes insipidus. Which of the following statements by the client requires further teaching? A. "I can drink up to 2 quarts of fluid a day." B. "I should expect to urinate frequently at night." C. "I may experience headaches." D. "I may experience a dry mouth."

A. "I can drink up to 2 quarts of fluid a day."

4. A nurse is caring for a client who has primary adrenal insufficiency. Which of the following findings should the nurse anticipate after an IV injection of ACTH 1.0 mg? A. Decrease in serum plasma cortisol B. Elevated fasting serum blood glucose C. Decrease in serum sodium D. Increase in urinary output

A. Decrease in serum plasma cortisol

3. A nurse is caring for a client who has syndrome of inappropriate antidiuretic hormone (SIADH). Which of the following findings should the nurse expect? (Select all that apply.) A. Decreased serum sodium B. Urine specific gravity 1.001 C. Serum osmolarity 230 mOsm/L D. Polyuria E. Increased thirst

A. Decreased serum sodium C. Serum osmolarity 230 mOsm/L

5. A nurse is providing discharge teaching to a client who experienced diabetic ketoacidosis. Which of the following should the nurse include in the teaching? (Select all that apply.) A. Drink 3 L of fluids daily. B. Monitor blood glucose every 4 hr when ill. C. Administer insulin as prescribed when ill. D. Notify the provider when blood glucose is 200 mg/dL. E. Report ketones in the urine after 24 hr of illness

A. Drink 3 L of fluids daily. B. Monitor blood glucose every 4 hr when ill. C. Administer insulin as prescribed when ill. E. Report ketones in the urine after 24 hr of illness

4. A nurse is presenting information to a group of clients about nutrition habits that prevent type 2 diabetes mellitus. Which of the following should the nurse include in the information? (Select all that apply.) A. Eat less meat and processed foods. B. Decrease intake of saturated fats. C. Increase daily fiber intake. D. Limit saturated fat intake to 15% of daily caloric intake. E. Include omega-3 fatty acids in the diet.

A. Eat less meat and processed foods. B. Decrease intake of saturated fats. C. Increase daily fiber intake. E. Include omega-3 fatty acids in the diet.

The client is diagnosed with hyperthyroidism. Which of the following clinical manifestations of hyperthyroidism should the nurse expect to find? (select all that apply.) A. Excessive Sweating B. Tremors C. Gastric Hypermotility D. Increased White Blood Cell Count E. Hypothermia F. Photophobia G. Exophthalmus

A. Excessive Sweating B. Tremors C. Gastric Hypermotility F. Photophobia G. Exophthalmus

3. A nurse is reviewing the health record of a client who has syndrome of inappropriate antidiuretic hormone (SIADH). Which of the following laboratory findings should the nurse anticipate? (Select all that apply.) A. Low serum sodium B. High serum potassium C. Decreased urine osmolality D. High urine sodium E. Increased urine-specific gravity

A. Low serum sodium D. High urine sodium E. Increased urine-specific gravity

5. A nurse in a provider's office is planning care for a client who has a new diagnosis of Graves' disease and a new prescription for methimazole (Tapazole). Which of the following should the nurse include in the plan of care? (Select all that apply.) A. Monitor CBC. B. Monitor triiodothyronine (T3). C. Inform the client that the medication should not be taken for more than 3 months. D. Advise the client to take the medication at the same time every day. E. Inform the client that an adverse effect of this medication is iodine toxicity.

A. Monitor CBC. B. Monitor triiodothyronine (T3). D. Advise the client to take the medication at the same time every day.

1. A nurse is caring for a client who has primary diabetes insipidus. Which of the following manifestations should the nurse expect to find? (Select all that apply.) A. Serum sodium of 155 mEq/L B. Fatigue C. Serum osmolality of 250 mOsm/L D. Polyuria E. Nocturia

A. Serum sodium of 155 mEq/L B. Fatigue D. Polyuria E. Nocturia

4. A nurse in an intensive care unit is admitting a client who has myxedema coma. Which of the following should the nurse anticipate in caring for this client? (Select all that apply.) A. Observe cardiac monitor for inverted T wave. B. Observe for evidence of urinary tract infection. C. Initiate IV fluids using 0.9% sodium chloride. D. Expect a prescription for levothyroxine (Synthroid) IV bolus. E. Provide warmth using a heating pad.

A. Observe cardiac monitor for inverted T wave. B. Observe for evidence of urinary tract infection. C. Initiate IV fluids using 0.9% sodium chloride. D. Expect a prescription for levothyroxine (Synthroid) IV bolus.

The provider prescribes medication for the client's hyperthyroidism. Which of the following medications blocks the synthesis of thyroid hormone? A. Propylthiouracil (PTU) B. Metoprolol (Lopressor) C. Amoxicillin (Amoxil) D. Docusate sodium (Colace)

A. Propylthiouracil (PTU)

4. A nurse is preparing to receive a client from the PACU who is postoperative following a thyroidectomy. The nurse should ensure that which of the following equipment is available? (Select all that apply.) A. Suction equipment B. Humidified air C. Flashlight D. Tracheostomy tray E. Oxygen delivery equipment

A. Suction equipment B. Humidified air D. Tracheostomy tray E. Oxygen delivery equipment

1. A nurse is reviewing the health record of a client who has hyperglycemic-hyperosmolar state (HHS). Which of the following data confirms this diagnosis? (Select all that apply.) A.Evidence of recent myocardial infraction B.BUN 35 mg/dL C.Takes a calcium channel blocker D.Age 77 years E.No insulin production

A.Evidence of recent myocardial infraction B.BUN 35 mg/dL C.Takes a calcium channel blocker D.Age 77 years

2. A nurse is reviewing the laboratory findings of a client who has suspected hyperthyroidism. An elevation of which of the following supports this diagnosis? A.Triodothyronine (T3) B.Vanillylmandelic acid (VMA) C. Adrenocorticotropic hormone (ACTH) D.Glycosylated hemoglobin (HbA1c)

A.Triodothyronine (T3) - T3 increases in a hyperthyroid state

Which patient action indicates good understanding of the nurse's teaching about administration of aspart (NovoLog) insulin? A. The patient avoids injecting the insulin into the upper abdominal area. B. The patient cleans the skin with soap and water before insulin administration. C. The patient stores the insulin in the freezer after administering the prescribed dose. D. The patient pushes the plunger down while removing the syringe from the injection site.

B - Cleaning the skin with soap and water or with alcohol is acceptable. Insulin should not be frozen. The patient should leave the syringe in place for about 5 seconds after injection to be sure that all the insulin has been injected. The upper abdominal area is one of the preferred areas for insulin injection.

The nurse identifies a need for additional teaching when the patient who is self-monitoring blood glucose A. Washes the puncture site using warm water and soap. B. Chooses a puncture site in the center of the finger pad. C. Hangs the arm down for a minute before puncturing the site. D. Says the result of 120 mg indicates good blood sugar control.

B - The patient is taught to choose a puncture site at the side of the finger pad because there are fewer nerve endings along the side of the finger pad. The other patient actions indicate that teaching has been effective.

Which information will the nurse include when teaching a 50-year-old patient who has type 2 diabetes about glyburide (Micronase, DiaBeta, Glynase)? A. Glyburide decreases glucagon secretion from the pancreas. B. Glyburide stimulates insulin production and release from the pancreas. C. Glyburide should be taken even if the morning blood glucose level is low. D. Glyburide should not be used for 48 hours after receiving IV contrast media.

B - The sulfonylureas stimulate the production and release of insulin from the pancreas. If the glucose level is low, the patient should contact the health care provider before taking the glyburide, because hypoglycemia can occur with this class of medication. Metformin should be held for 48 hours after administration of IV contrast media, but this is not necessary for glyburide. Glucagon secretion is not affected by glyburide.

1. A client asks a nurse why the provider bases his medication regimen on his HbA1c instead of his log of morning fasting blood glucose results. Which of the following is an appropriate response by the nurse? A. "HbA1c measures how well insulin is regulating your blood glucose between meals." B. "HbA1c indicates how well your blood glucose has been regulated over the past 3 months." HbA1c measures the client's blood glucose control over the past 2 to 3 months. C. "A test of HbA1c is the first test to determine if an individual has diabetes." D. "A test of HbA1c determines if the dosage of insulin needs to be adjusted."

B. "HbA1c indicates how well your blood glucose has been regulated over the past 3 months." HbA1c measures the client's blood glucose control over the past 2 to 3 months.

2. A nurse is preparing to administer a morning dose of aspart insulin (NovoLog) to a client who has type 1 diabetes mellitus. Which of the following is an appropriate action by the nurse? A. Check the client's blood glucose immediately after breakfast. B. Administer the insulin when breakfast arrives. C. Hold breakfast for 1 hr after insulin administration. D. Clarify the prescription because insulin should not be administered at this time.

B. Administer the insulin when breakfast arrives.

2. A nurse is caring for a client who has diabetes insipidus. Which of the following urinalysis laboratory findings should the nurse anticipate? A. Absence of glucose B. Decreased specific gravity C. Presence of ketones D. Presence of red blood cells

B. Decreased specific gravity

1. A nurse in a provider's office is reviewing the health record of a client who is being evaluated for Graves' disease. Which of the following is an expected laboratory finding for this client? A. Decreased thyrotropin receptor antibodies B. Decreased thyroid stimulating hormone C. Decreased free thyroxine index D. Decreased triiodothyronine

B. Decreased thyroid stimulating hormone

2. A nurse is assessing a client who has diabetic ketoacidosis and ketones in the urine. Which of the following are expected findings? (Select all that apply.) A. Weight gain B. Fruity odor of breath C. Abdominal pain D. Kussmaul respirations E. Metabolic acidosis

B. Fruity odor of breath C. Abdominal pain D. Kussmaul respirations E. Metabolic acidosis

2. A nurse is reviewing the clinical manifestations of hyperthyroidism with a client. Which of the following findings should the nurse include? (Select all that apply.) A. Dry skin B. Heat intolerance C. Constipation D. Palpitations E. Weight loss F. Bradycardia

B. Heat intolerance D. Palpitations E. Weight loss

3. A nurse is reinforcing teaching with a client who has been prescribed levothyroxine (Synthroid) to treat hypothyroidism. Which of the following should the nurse include in the teaching? (Select all that apply.) A. Weight gain is expected while taking this medication. B. Medication should not be discontinued without the advice of the provider. C. Follow-up serum TSH levels should be obtained. D. Take the medication on an empty stomach. E. Use fiber laxatives for constipation.

B. Medication should not be discontinued without the advice of the provider. C. Follow-up serum TSH levels should be obtained. D. Take the medication on an empty stomach.

2. A nurse is collecting an admission history from a female client who has hypothyroidism. Which of the following findings are expected with this condition? (Select all that apply.) A. Diarrhea B. Menorrhagia C. Dry skin D. Increased libido E. Hoarseness

B. Menorrhagia C. Dry skin E. Hoarseness

A nurse on the surgical unit is preparing to care for a client 12 hr post-total thyroidectomy. Which of the following interventions should the nurse anticipate implementing? (select all that apply) A. Restrict the client from speaking B. Monitor vital sign every 4 hour C. Keep the client in high-fowler's position D. Administer mild analgesics as prescribed E. Support the neck while client coughs and deep breathes every 2 hour

B. Monitor vital sign every 4 hour C. Keep the client in high-fowler's position D. Administer mild analgesics as prescribed E. Support the neck while client coughs and deep breathes every 2 hour

A nurse caring for a client recognizes that the client TSH is reliable indicator of the efficacy of the levothyroxine sodium because the TSH will A. Have a value of zero when when an euthyroid state is re-established B. Return to its expected reference range when an euthyroid state is reestablished C. Increase above its expected reference range when a therapeutic medication level is reached D. Decrease below it's expected reference range when a therapeutic medication level is reached

B. Return to its expected reference range when an euthyroid state is reestablished

3. A nurse is reviewing laboratory reports of a client who has hyperglycemic-hyperosmolar state (HHS). Which of the following is an expected finding? A. Serum pH 7.2 B. Serum osmolarity 350 mOsm/L C. Serum potassium 3.8 mg/dL D. Serum creatinine 0.8 mg/dL

B. Serum osmolarity 350 mOsm/L

3. A nurse is providing instructions to a client who has Graves' disease and has a new prescription for propranolol (Inderal). Which of the following information should the nurse include? A. An adverse effect of this medication is jaundice. B. Take your pulse before each dose. C. The purpose of this medication is to decrease production of thyroid hormone. D. You should stop taking this medication if you have a sore throat.

B. Take your pulse before each dose.

Which statement by a nurse to a patient newly diagnosed with type 2 diabetes is correct? A. Insulin is not used to control blood glucose in patients with type 2 diabetes. B. Complications of type 2 diabetes are less serious than those of type 1 diabetes. C. Changes in diet and exercise may control blood glucose levels in type 2 diabetes. D. Type 2 diabetes is usually diagnosed when the patient is admitted with a hyperglycemic coma.

C - For some patients with type 2 diabetes, changes in lifestyle are sufficient to achieve blood glucose control. Insulin is frequently used for type 2 diabetes, complications are equally severe as for type 1 diabetes, and type 2 diabetes is usually diagnosed with routine laboratory testing or after a patient develops complications such as frequent yeast infections.

When a patient who takes metformin (Glucophage) to manage type 2 diabetes develops an allergic rash from an unknown cause, the health care provider prescribes prednisone (Deltasone). The nurse will anticipate that the patient may A. Need a diet higher in calories while receiving prednisone. B. Develop acute hypoglycemia while taking the prednisone. C. Require administration of insulin while taking prednisone. D. Have rashes caused by metformin-prednisone interactions.

C - Glucose levels increase when patients are taking corticosteroids, and insulin may be required to control blood glucose. Hypoglycemia is not a side effect of prednisone. Rashes are not an adverse effect caused by taking metformin and prednisone simultaneously. The patient may have an increased appetite when taking prednisone, but will not need a diet that is higher in calories.

A 48-year-old male patient screened for diabetes at a clinic has a fasting plasma glucose level of 120 mg/dL (6.7 mmol/L). The nurse will plan to teach the patient about A. Self-monitoring of blood glucose. B. Using low doses of regular insulin. C. Lifestyle changes to lower blood glucose. D. Effects of oral hypoglycemic medications.

C - The patient's impaired fasting glucose indicates prediabetes, and the patient should be counseled about lifestyle changes to prevent the development of type 2 diabetes. The patient with prediabetes does not require insulin or oral hypoglycemics for glucose control and does not need to self-monitor blood glucose.

The nurse is assessing a 22-year-old patient experiencing the onset of symptoms of type 1 diabetes. Which question is most appropriate for the nurse to ask? A. "Are you anorexic?" B. "Is your urine dark colored?" C. "Have you lost weight lately?" D. "Do you crave sugary drinks?"

C - Weight loss occurs because the body is no longer able to absorb glucose and starts to break down protein and fat for energy. The patient is thirsty but does not necessarily crave sugar-containing fluids. Increased appetite is a classic symptom of type 1 diabetes. With the classic symptom of polyuria, urine will be very dilute.

4. A nurse is assessing a client who has SIADH. Which of the following findings indicate the client is experiencing a complication? A. Decreased central venous pressure (CVP) B. Increased urine output C. Distended neck veins D. Extreme thirst

C. Distended neck veins

3. A nurse is preparing to administer the morning doses of glargine (Lantus) insulin and regular (Humulin R) insulin to a client who has a blood glucose of 278 mg/dL. Which of the following is an appropriate nursing action? A. Draw up the regular insulin and then the glargine insulin in the same syringe. B. Draw up the glargine insulin then the regular insulin in the same syringe. C. Draw up and administer regular and glargine insulin in separate syringes. D. Administer the regular insulin, wait 1 hr, and then administer the glargine insulin.

C. Draw up and administer regular and glargine insulin in separate syringes.

1. A nurse in a provider's office is reviewing the laboratory findings of a client who is being evaluated for primary hypothyroidism. Which of the following laboratory findings is expected for a client who has this condition? A. Serum T4 10 mcg/dL B. Serum T3 200 ng/dL C. Hematocrit 34% D. Serum cholesterol 180 mg/dL

C. Hematocrit 34%

5. A nurse is teaching foot care to a client who has diabetes mellitus. Which of the following information should the nurse include in the teaching? (Select all that apply.) A. Remove calluses using over-the-counter remedies. B. Apply lotion between toes. C. Perform nail care after bathing. D. Trim toenails straight across. E. Wear closed-toe shoes

C. Perform nail care after bathing. D. Trim toenails straight across. E. Wear closed-toe shoes

1. A nurse is caring for a client who has blood glucose of 52 mg/dL. The client is lethargic but arousable. Which of the following actions should the nurse perform first? A. Recheck blood glucose in 15 min. B. Provide a carbohydrate and protein food. C. Provide 4 oz grape juice. D. Report findings to the provider.

C. Provide 4 oz grape juice.

4. A nurse is preparing to administer IV fluids to a client who has diabetic ketoacidosis. Which of the following is an appropriate nursing action? A. Administer an IV infusion of regular insulin at 0.3 unit/kg/hr. B. Administer an IV infusion of 0.45% sodium chloride. C. Rapidly administer an IV infusion of 0.9% sodium chloride. D. Add glucose to the IV infusion when serum glucose is 350 mg/dL

C. Rapidly administer an IV infusion of 0.9% sodium chloride.

Which of the following should the nurse reinforce in regard to taking levothyroxine? (select all that apply) A. Use fiber laxative for constipation B. Expect to take this medication for 3-6 months C. Return to clinic for follow up lab tests of serum TSH D. Take medication 30 min before bedtime E. Side effect such as nervousness, heat intolerance, and diarrhea

C. Return to clinic for follow up lab tests of serum TSH E. Side effect such as nervousness, heat intolerance, and diarrhea

6. A nurse is assessing a client who is 12 hr postoperative following a thyroidectomy. Which of the following findings are indicative of thyroid crisis? (Select all that apply.) A. Bradycardia B. Hypothermia C. Tremors D. Abdominal pain E. Mental confusion

C. Tremors D. Abdominal pain E. Mental confusion

A nurse is caring for a client with hyperparathyroidism who has undergone surgical removal of the parathyroid glands. The knows that improvement in the client's condition is indicated by a decrease in serum?

Calcium

A hospitalized diabetic patient received 38 U of NPH insulin at 7:00 AM. At 1:00 PM, the patient has been away from the nursing unit for 2 hours, missing the lunch delivery while awaiting a chest x-ray. To prevent hypoglycemia, the best action by the nurse is to A. Save the lunch tray for the patient's later return to the unit. B. Ask that diagnostic testing area staff to start a 5% dextrose IV. C. Send a glass of milk or orange juice to the patient in the diagnostic testing area. D. Request that if testing is further delayed, the patient be returned to the unit to eat.

D - Consistency for mealtimes assists with regulation of blood glucose, so the best option is for the patient to have lunch at the usual time. Waiting to eat until after the procedure is likely to cause hypoglycemia. Administration of an IV solution is unnecessarily invasive for the patient. A glass of milk or juice will keep the patient from becoming hypoglycemic but will cause a rapid rise in blood glucose because of the rapid absorption of the simple carbohydrate in these items.

A patient with type 2 diabetes is scheduled for a follow-up visit in the clinic several months from now. Which test will the nurse schedule to evaluate the effectiveness of treatment for the patient? A. Urine dipstick for glucose B. Oral glucose tolerance test C. Fasting blood glucose level D. Glycosylated hemoglobin level

D - The glycosylated hemoglobin (A1C or HbA1C) test shows the overall control of glucose over 90 to 120 days. A fasting blood level indicates only the glucose level at one time. Urine glucose testing is not an accurate reflection of blood glucose level and does not reflect the glucose over a prolonged time. Oral glucose tolerance testing is done to diagnose diabetes, but is not used for monitoring glucose control once diabetes has been diagnosed.

The nurse has been teaching a patient with type 2 diabetes about managing blood glucose levels and taking glipizide (Glucotrol). Which patient statement indicates a need for additional teaching? A. "If I overeat at a meal, I will still take the usual dose of medication." B. "Other medications besides the Glucotrol may affect my blood sugar." C. "When I am ill, I may have to take insulin to control my blood sugar." D. "My diabetes won't cause complications because I don't need insulin."

D - The patient should understand that type 2 diabetes places the patient at risk for many complications and that good glucose control is as important when taking oral agents as when using insulin. The other statements are accurate and indicate good understanding of the use of glipizide.

azotemia

an accumulation of nitrogenous waste products such as blood urea nitrogen (BUN) and creatinine (Lewis page 1069)

A nurse admits that a client with newly diagnosed DM. When reviewing the client's lab work, the nurse notes that the result consistent with diabetic ketoacidosis is what?

bicarbonate level of 12.

Oliguria

less than 400 mL of urine in 24 hours (Lewis page 1071)

A nurse is instructing a client with DM II regarding the patho of the client's condition. What statement made by the client shows that learning is taking place?

my cells are resistant to the effects of insulin."

continuous ambulatory peritoneal dialysis (CAPD)

dialysis carried out manually by exchanging 1.5 to 3 L of peritoneal dialysate usually four times daily with dwell times of 4 to 10 hours; an indwelling catheter permits fluid to drain into and out of the peritoneal cavity by gravity (Lewis page 1086)

automated peritoneal dialysis (ADP)

dialysis machine that controls the fill, dwell, and drain phases, and cycles four to eight exchanges per night with 1 to 2 hours per exchange (Lewis page 1086)

Hemodialysis (HD)

dialysis that uses an artificial membrane (usually made of cellulose-based or synthetic materials) as the semipermeable membrane through which the patient's blood circulates; impurities or wastes are removed from the blood (Lewis page 1084)

peritoneal dialysis (PD)

dialysis with the use of the peritoneal membrane as the semipermeable membrane; performed to correct an imbalance of fluid or of electrolytes in the blood or to remove toxins, drugs, or other wastes normally excreted by the kidney (Lewis page 1084)

A nurse is assessing a client with Grave's disease. The nurse should expect the client to report what?

difficulty sleeping

The nurse is teaching a client with DM I about early manifestations of hypoglycemia. What should the nurse be sure to include in her teaching?

drowsiness

arteriovenous grafts (AVGs)

fistula made of synthetic materials that forms a "bridge" between the arterial and venous blood supplies (Lewis page 1088)

Dialysis

technique in which substances move from the blood through a semipermeable membrane and into a dialysis solution; used to correct fluid and electrolyte imbalances and to remove waste products in renal failure (Lewis page 1084)

chronic kidney disease (CKD)

the presence of kidney damage for at least 3 months with functional or structural abnormalities, with or without decreased glomerular filtration rate; can also be a glomerular filtration rate <60 mL/min/1.73 m2 for greater than 3 months, with or without damage to the kidney (Lewis page 1075)

acute kidney injury (AKI)

the term used to encompass the entire scope of the syndrome, ranging from a slight deterioration in kidney function to severe impairment. Characterized by a rapid loss of kidney function. This loss is accompanied by a rise in serum creatinine and/or a reduction in urine output. (Lewis page 1069)

Symptoms of Cushing Syndrome

thinning of hair, red cheeks, acne, buffalo hump, moon face, supraclavicular fat pad, increased body/facial hair, weight gain, purple striae on pendulous abdomen, echymosis from easy bruising, thin extremities w/ muscle atrophy, thin skin/subcutaneous tissue, slow wound healing

arteriovenous fistula (AVF)

type of vascular access created most commonly in the forearm with an anastomosis between an artery (usually radial or ulnar) and a vein (usually cephalic) (Lewis page 1087)

The nurse is performing discharge education for a patient who was admitted for acute hypothyroidism. The patient is undergoing thyroid hormone therapy for the first time. What statement by the patient to the nurse confirms that discharge teaching was effective? 1 "I should take my levothyroxine every morning before eating my breakfast." 2 "I should only follow up with my doctor if I start having shortness of breath." 3 "I should keep the air conditioning a few degrees colder to help me with sweating." 4 "I should limit the amount of fiber I am eating to help keep me from getting constipated."

1 "I should take my levothyroxine every morning before eating my breakfast." A patient with a new diagnosis of hypothyroidism should be taught how to manage hypothyroidism, including taking the thyroid hormone in the morning before food. Patients with hypothyroidism need to be taught about the importance of regular follow-up care, not just when they are having abnormal symptoms. Patents with hypothyroidism should be taught to keep the environment warm and comfortable because of cold intolerance. Patients with hypothyroidism should increase the amount of fiber in their diet to prevent constipation; they should not limit the amount of fiber.

The nurse expects that which drug will be prescribed for the treatment of a patient diagnosed with hyperthyroidism, asthma, and heart disease? 1 Atenolol 2 Methimazole 3 Lugol's solution 4 Propylthiouracil

1 Atenolol Atenolol, a β-Adrenergic blocker, is prescribed to control the stimulation of the sympathetic nervous system that often occurs with hyperthyroidism. Atenolol manages tachycardia, nervousness, irritability, and tremors. It is considered the drug of choice for treating a patient diagnosed with hyperthyroidism, asthma, and heart disease. Methimazole is used to treat hyperthyroidism; however, it is not the drug of choice for patients with concurrent diagnoses of asthma and heart disease. Lugol's solution is an antithyroid drug that is used in treatment of thyrotoxicosis. Propylthiouracil, although appropriate for the treatment of hyperthyroidism, is not the drug of choice for a patient with concurrent diagnoses of asthma and heart disease.

Activity intolerance in a patient with hypothyroidism is related to what? 1 Fatigue 2 Diarrhea 3 Weight loss 4 Nervousness

1 Fatigue Activity intolerance in a patient with hypothyroidism is related to weakness and fatigue. Patients with hyperthyroidism, not hypothyroidism, experience weight loss, diarrhea, and nervousness.

The nurse is caring for a patient who underwent removal of the thyroid gland (thyroidectomy) three days ago. The patient's serum chemistries reveal calcium of 3.2 mg/dL, potassium of 3.9 mEq/L, and phosphorus of 4.0 mg/dL. What condition do these findings indicate? 1 Hypocalcemia 2 Hypercalcemia 3 Hyperkalemia 4 Hypophosphatemia

1 Hypocalcemia Hypocalcemia is a low serum calcium level. Surgical removal of the thyroid gland may also include removal of the parathyroid gland. This results in a deficiency of parathyroid hormone, which controls serum calcium by regulating absorption of calcium from the gastrointestinal tract, mobilizing calcium in bones, and excreting calcium in breast milk, feces, sweat, and urine. The normal serum calcium level ranges from 9.0 to 11.5 mg/dL. Potassium is within normal limits (3.5 to 5 mEq/L), and phosphorus is also within normal limits (2.8 to 4.5 mg/dL).

The nurse teaches a patient about diagnostic tests for ascertaining the presence of Cushing syndrome. Which advice would the nurse tell the patient to do to confirm the diagnosis? Select all that apply. 1 "Check plasma adrenocorticotropic hormone (ACTH) levels." 2 "Take blood chemistries for sodium, potassium, and calcium." 3 "Opt for a complete blood count (CBC) with RBC differential." 4 "Collect a 24-hour urine sample for testing free cortisol and 17-ketosteroids." 5 "Undergo computed tomography (CT) scan or magnetic resonance imaging (MRI)."

1 "Check plasma adrenocorticotropic hormone (ACTH) levels." 4 "Collect a 24-hour urine sample for testing free cortisol and 17-ketosteroids." 5 "Undergo computed tomography (CT) scan or magnetic resonance imaging (MRI)." Checking plasma adrenocorticotropic (ACTH) hormone levels will help assess the underlying cause of Cushing syndrome because a high or normal ACTH level indicates Cushing syndrome. A urine cortisol level higher than the normal range of 80 to 120 mcg/24 hr is an indicator of Cushing syndrome. Both a computed tomography (CT) scan and a magnetic resonance imaging (MRI) of the pituitary and adrenal glands are used to detect Cushing syndrome. Blood chemistries for sodium, potassium, and glucose are also a part of the diagnostic tests for Cushing syndrome. A complete blood count (CBC) with WBC, not RBC, differential is usually performed as part of the diagnostic tests for Cushing syndrome.

The nurse is caring for a patient being treated for acute thyrotoxicosis. What are the nursing interventions for this patient exhibiting exophthalmos? Select all that apply. 1 Apply artificial tears. 2 Tape the eyelids lightly for sleeping, if needed. 3 Ask the patient to exercise the intraocular muscles. 4 Place the patient in a supine position. 5 Avoid elevating the patient's head.

1 Apply artificial tears. 2 Tape the eyelids lightly for sleeping, if needed. 3 Ask the patient to exercise the intraocular muscles. Nursing interventions for the patient exhibiting exophthalmos include application of artificial tears to soothe and moisten conjunctival membranes, to relieve eye discomfort, and to prevent corneal ulceration. If the eyelids cannot be closed, the nurse should lightly tape them shut to help the patient sleep. To maintain flexibility, the patient must be taught to exercise the intraocular muscles several times a day by turning the eyes in the complete range of motion. The patient should sit upright as much as possible. The head must be elevated to promote fluid drainage from the periorbital area.

Which nursing interventions are appropriate when providing care to a patient that is recovering from a thyroidectomy? Select all that apply. 1 Assessing for tetany 2 Monitoring vital signs 3 Monitoring potassium levels 4 Assessing the patient every two hours on the first postoperative day 5 Placing the patient in a high Fowler's position

1 Assessing for tetany 2 Monitoring vital signs 4 Assessing the patient every two hours on the first postoperative day Postoperative nursing interventions that are appropriate for a patient after a thyroidectomy include assessing for tetany, monitoring vital signs, and assessing the patient every two hours on the first postoperative day for hemorrhage and tracheal compression. The nurse should monitor calcium levels, not potassium levels. The nurse should place the patient in a semi-Fowler's position to reduce swelling and edema in the neck area. Sandbags or pillows may be used to support the head or neck.

Which clinical manifestations does the nurse expect during the assessment of a hospitalized patient experiencing exophthalmos? Select all that apply. 1 Dyspnea 2 Celiac disease 3 Cardiac hypertrophy 4 Distended abdomen 5 Bounding, rapid pulse

1 Dyspnea 3 Cardiac hypertrophy 5 Bounding, rapid pulse Exophthalmos is a classic finding in Graves' disease, which is caused by hyperthyroidism. Clinical manifestations anticipated by the nurse upon assessment include a bounding, rapid pulse; cardiac hypertrophy; and dyspnea. Clinical symptoms such as celiac disease and distended abdomen are associated with hypothyroidism.

Which clinical manifestations does the nurse expect to assess in a patient that is diagnosed with hyperthyroidism? Select all that apply. 1 Weight loss 2 Protrusion of the eye balls 3 Thick, cold, and dry skin 4 Elevated blood pressure 5 Purplish red marks on abdomen

1 Weight loss 2 Protrusion of the eye balls 4 Elevated blood pressure Weight loss, protrusion of the eyeballs, and elevated blood pressure are clinical manifestations of hyperthyroidism. Weight loss and hypertension are due to increases in metabolic demands; protrusion of the eyeballs is due in part to accumulation of fluid in the eyes. Thick, cold, and dry skin are symptoms of hypothyroidism. Purplish red marks on the abdomen are seen in Cushing syndrome.

The nurse is monitoring a client with Graves' disease for signs of thyrotoxic crisis (thyroid storm). Which signs or symptoms, if noted in the client, will alert the nurse to the presence of this crisis? 1. Fever and tachycardia 2. Pallor and tachycardia 3. Agitation and bradycardia 4. Restlessness and bradycardia

1. Fever and tachycardia Rationale: Thyrotoxic crisis (thyroid storm) is an acute, potentially life-threatening state of extreme thyroid activity that represents a breakdown in the body's tolerance to a chronic excess of thyroid hormones. The clinical manifestations include fever with temperatures greater than 100°F, severe tachycardia, flushing and sweating, and marked agitation and restlessness. Delirium and coma can occur.

A client arrives in the hospital emergency department in an unconscious state. As reported by the spouse, the client has diabetes mellitus and began to show symptoms of hypoglycemia. A blood glucose level is obtained for the client, and the result is 40 mg/dL. Which medication should the nurse anticipate to be prescribed for the client? 1. Glucagon 2. Humulin N insulin 3. Humulin R insulin 4. Glyburide (DiaBeta)

1. Glucagon Rationale: A blood glucose level lower than 50 mg/dL (2.85 mmol/L) is considered to be critically low. Glucagon is used to treat hypoglycemia because it increases blood glucose levels. Insulin would lower the client's blood glucose and would not be an appropriate treatment for hypoglycemia. Glyburide and metformin are oral hypoglycemic agents used to treat type 2 diabetes mellitus and would not be given to a client with hypoglycemia. In addition, an oral medication would not be administered to an unconscious client.

A client with a diagnosis of Addisonian crisis is being admitted to the intensive care unit. Which findings will the interprofessional health care team focus on? Select all that apply. 1. Hypotension 2. Leukocytosis 3. Hyperkalemia 4. Hypercalcemia 5. Hypernatremia

1. Hypotension 3. Hyperkalemia Rationale: In Addison's disease, also known as adrenal insufficiency, destruction of the adrenal gland leads to decreased production of adrenocortical hormones, including the glucocorticoid cortisol and the mineralocorticoid aldosterone. Addisonian crisis, also known as acute adrenal insufficiency, occurs when there is extreme physical or emotional stress and lack of sufficient adrenocortical hormones to manage the stressor. Addisonian crisis is a life-threatening emergency. One of the roles of endogenous cortisol is to enhance vascular tone and vascular response to the catecholamines epinephrine and norepinephrine. Hypotension occurs when vascular tone is decreased and blood vessels cannot respond to epinephrine and norepinephrine. The role of aldosterone in the body is to support the blood pressure by holding salt and water and excreting potassium. When there is insufficient aldosterone, salt and water are lost and potassium builds up; this leads to hypotension from decreased vascular volume, hyponatremia, and hyperkalemia. The remaining options are not associated with Addisonian crisis.

The nurse is caring for a client who has had an adrenalectomy and is monitoring the client for signs of adrenal insufficiency. Which signs and symptoms indicate adrenal insufficiency in this client? 1. Hypotension and fever 2. Mental status changes and hypertension 3. Subnormal temperature and hypotension 4. Complaints of weakness and hypertension

1. Hypotension and fever Rationale: The nurse should be alert to signs and symptoms of adrenal insufficiency after adrenalectomy. These signs and symptoms include weakness, hypotension, fever, and mental status changes. The remaining options are incorrect.

The nurse is admitting a client who is diagnosed with syndrome of inappropriate antidiuretic hormone secretion (SIADH) and has serum sodium of 118 mEq/L (118 mmol/L). Which health care provider prescriptions should the nurse anticipate receiving? Select all that apply. 1. Initiate an infusion of 3% NaCl. 2. Administer intravenous furosemide. 3. Restrict fluids to 800 mL over 24 hours. 4. Elevate the head of the bed to high Fowler's. 5. Administer a vasopressin antagonist as prescribed.

1. Initiate an infusion of 3% NaCl. 3. Restrict fluids to 800 mL over 24 hours. 5. Administer a vasopressin antagonist as prescribed. Rationale: Clients with SIADH experience excess secretion of antidiuretic hormone (ADH), which leads to excess intravascular volume, a declining serum osmolarity, and dilutional hyponatremia. Management is directed at correcting the hyponatremia and preventing cerebral edema. Hypertonic saline is prescribed when the hyponatremia is severe, less than 120 mEq/L (120 mmol/L). An intravenous (IV) infusion of 3% saline is hypertonic. Hypertonic saline must be infused slowly as prescribed and an infusion pump must be used. Fluid restriction is a useful strategy aimed at correcting dilutional hyponatremia. Vasopressin is an ADH; vasopressin antagonists are used to treat SIADH. Furosemide may be used to treat extravascular volume and dilutional hyponatremia in SIADH, but it is only safe to use if the serum sodium is at least 125 mEq/L (125 mmol/L). When furosemide is used, potassium supplementation should also occur and serum potassium levels should be monitored. To promote venous return, the head of the bed should not be raised more than 10 degrees for the client with SIADH. Maximizing venous return helps to avoid stimulating stretch receptors in the heart that signal to the pituitary that more ADH is needed.

The nurse is developing a plan of care for a client with Cushing's syndrome. The nurse documents a client problem of excess fluid volume. Which nursing actions should be included in the care plan for this client? Select all that apply. 1. Monitor daily weight. 2. Monitor intake and output. 3. Assess extremities for edema. 4. Maintain a high-sodium diet. 5. Maintain a low-potassium diet.

1. Monitor daily weight. 2. Monitor intake and output. 3. Assess extremities for edema. Rationale: The client with Cushing's syndrome and a problem of excess fluid volume should be on daily weights and intake and output and have extremities assessed for edema. He or she should be maintained on a high-potassium, low-sodium diet. Decreased sodium intake decreases renal retention of sodium and water.

The nurse is developing a plan of care for a client with Addison's disease. The nurse has identified a problem of risk for deficient fluid volume and identifies nursing interventions that will prevent this occurrence. Which nursing interventions should the nurse include in the plan of care? Select all that apply. 1. Monitor for changes in mentation. 2. Encourage an intake of low-protein foods. 3. Encourage an intake of low-sodium foods. 4. Encourage fluid intake of at least 3000 mL per day. 5. Monitor vital signs, skin turgor, and intake and output.

1. Monitor for changes in mentation. 4. Encourage fluid intake of at least 3000 mL per day. 5. Monitor vital signs, skin turgor, and intake and output. Rationale: The client at risk for deficient fluid volume should be encouraged to eat regular meals and snacks and to increase intake of sodium, protein, and complex carbohydrates and fluids. Oral replacement of sodium losses is necessary, and maintenance of adequate blood glucose levels is required. Mentation, vital signs, skin turgor and intake and output should be monitored for signs of fluid volume deficit.

A nurse is reviewing the health care provider's prescriptions for a client diagnosed with hypothyroidism. Which medication prescription should the nurse question and verify? 1. Morphine sulfate 2. Docusate sodium (Colace) 3. Acetaminophen (Tylenol) 4. Levothyroxine sodium (Synthroid)

1. Morphine sulfate

The nurse is completing an assessment on a client who is being admitted for a diagnostic workup for primary hyperparathyroidism. Which client complaint would be characteristic of this disorder? Select all that apply. 1. Polyuria 2. Headache 3. Bone pain 4. Nervousness 5. Weight gain

1. Polyuria 3. Bone pain Rationale: The role of parathyroid hormone (PTH) in the body is to maintain serum calcium homeostasis. In hyperparathyroidism, PTH levels are high, which causes bone resorption (calcium is pulled from the bones). Hypercalcemia occurs with hyperparathyroidism. Elevated serum calcium levels produce osmotic diuresis and thus polyuria. This diuresis leads to dehydration (weight loss rather than weight gain). Loss of calcium from the bones causes bone pain. Options 2, 4, and 5 are not associated with hyperparathyroidism. Some gastrointestinal symptoms include anorexia, nausea, vomiting, and constipation.

The nurse is monitoring a client newly diagnosed with diabetes mellitus for signs of complications. Which sign, if exhibited in the client, would indicate hyperglycemia? 1. Polyuria 2. Diaphoresis 3. Hypertension 4. Increased pulse rate

1. Polyuria Rationale: Chronic hyperglycemia, resulting from poor glycemic control, contributes to the microvascular and macrovascular complications of diabetes mellitus. Classic symptoms of hyperglycemia include polydipsia, polyuria, and polyphagia. Diaphoresis may occur in hypoglycemia. Hypoglycemia is an acute complication of diabetes mellitus; however, it does not predispose a client to the chronic complications of diabetes mellitus. Therefore, option 2 can be eliminated because this finding is characteristic of hypoglycemia. Options 3 and 4 are not associated with diabetes mellitus.

The nurse is caring for a patient with pheochromocytoma. Which intervention would help prevent the sudden release of catecholamines and sudden hypertension? 1 Nourishing the patient with a healthy diet 2 Avoiding palpations of the patient's abdomen 3 Advising the patient to rise slowly from the bed 4 Administering α- and β-blockers preoperatively to the patient

2 Avoiding palpations of the patient's abdomen The nurse should avoid palpating the abdomen of a patient with suspected pheochromocytoma because the action may cause the sudden release of the catecholamines and severe hypertension. A healthy diet promotes the overall health of the patient. Advising the patient to rise slowly from the bed helps prevent orthostatic hypotension. Administering α- and β-blockers preoperatively helps prevent an intraoperative hypertensive crisis.

A patient was admitted to an inpatient unit for general weakness. The patient had laboratory tests completed, and the nurse is reviewing the results in the electronic medical record. The primary health care provider suspects hypothyroidism. The nurse recognizes that the patient is experiencing primary hypothyroidism. Which laboratory values support the suspicion of primary hypothyroidism? 1 Low thyroid-stimulating hormone level, low thyroxine level 2 High thyroid-stimulating hormone level, low thyroxine level 3 Low thyroid-stimulating hormone level, low basal metabolic rate 4 Low thyroid-stimulating hormone level, high basal metabolic rate

2 High thyroid-stimulating hormone level, low thyroxine level Primary hypothyroidism is caused by destruction of thyroid tissue or defective hormone synthesis. It is characterized by a high thyroid-stimulating hormone (TSH) level and a low thyroxine level. A low TSH level and low thyroxine level support secondary hypothyroidism. A low TSH level and a low basal metabolic rate (BMR) support secondary hypothyroidism. A low TSH and high BMR indicate hyperthyroidism

The patient is brought to the emergency department following a car accident and is wearing medical identification that says the patient has Addison's disease. What should the nurse expect to be included in the collaborative care of this patient? 1 Low-sodium diet 2 Increased glucocorticoid replacement 3 Suppression of pituitary adrenocorticotropic hormone (ACTH) synthesis 4 Elimination of mineralocorticoid replacement

2 Increased glucocorticoid replacement The patient with Addison's disease needs lifelong glucocorticoid and mineralocorticoid replacement and has an increased need with illness, injury, or stress, as this patient is experiencing. The patient with Addison's also may need a high-sodium diet. Suppression of pituitary ACTH synthesis is done for Cushing's syndrome. Elimination of mineralocorticoid replacement cannot be done for Addison's disease.

The nurse creates a plan of care for a patient with a pheochromocytoma. What is an appropriate expected outcome for the patient? 1 Verbalizing coping mechanisms 2 Maintaining a normotensive state 3 Maintaining a decreased activity level 4 Demonstrating compliance with dietary instructions

2 Maintaining a normotensive state A pheochromocytoma is a benign tumor of the adrenal gland, the major manifestation of which is severe hypertension due to excessive secretion of catecholamines, such as epinephrine. Therefore, the priority goal for this patient would be to maintain a normal blood pressure, or a normotensive state. At least 10% to 30% of patients require antihypertensive medication after the surgery. If the blood pressure returns to a normotensive state, the need to verbalize coping mechanisms, a decreased activity level, and compliance with dietary restrictions do not apply.

What should the nurse include in dietary instructions provided to a patient who is diagnosed with hyperthyroidism? Select all that apply. 1 Eat a high-fiber diet. 2 Consume a high-calorie diet. 3 Eat snacks high in protein. 4 Avoid caffeinated beverages. 5 Decrease the intake of carbohydrates.

2 Consume a high-calorie diet. 3 Eat snacks high in protein. 4 Avoid caffeinated beverages. A diet high in calories and protein is encouraged. Caffeinated beverages should be avoided. High-fiber foods should be avoided, not encouraged, because they can further stimulate the already hyperactive gastrointestinal tract. The patient should increase intake of carbohydrate-rich foods to compensate for the increased metabolism. This provides energy and decreases the use of body-stored protein.

A nurse is caring for a patient who underwent subtotal thyroidectomy because of the overproduction and release of thyroid hormone. Postoperative nursing interventions are important to prevent complications after surgery. Which nursing interventions should the nurse implement for safe, effective care? Select all that apply. 1 Monitor vital signs and potassium levels. 2 Control postoperative pain by administering medication. 3 Place the patient supine and support the head with pillows. 4 Assess for signs of tetany secondary to hypoparathyroidism. 5 Assess the patient every two hours for signs of bleeding or tracheal compression.

2 Control postoperative pain by administering medication. 4 Assess for signs of tetany secondary to hypoparathyroidism. 5 Assess the patient every two hours for signs of bleeding or tracheal compression. Nursing interventions after a thyroidectomy are important to prevent complications, such as airway obstruction. These interventions include controlling pain with medication; assessing for signs of tetany (i.e., tingling in toes, fingers, and around the mouth, Trousseau sign, and Chvostek sign); and assessing the patient every two hours for signs of bleeding and tracheal compression. Monitoring vital signs is important, but monitoring potassium levels is not; the calcium levels should be monitored. The patient should be placed in a semi-Fowler's position, not supine, with the head supported with pillows.

A patient with an endocrine disorder is prescribed corticosteroids. Which parameters should the nurse monitor for early detection of side effects? Select all that apply. 1 Increased risk for ulcers 2 Decreased bone density 3 Increased potassium levels 4 Decreased risk for infections 5 Increased level of blood pressure

2 Decreased bone density 5 Increased level of blood pressure Decreased bone density due to the prolonged use of corticosteroids may lead to bone weakness; therefore, the patient is advised to take calcium supplements. Corticosteroids may increase the blood pressure by causing a decrease in the level of potassium and promoting retention of sodium. The drug may increase the risk of ulcers but only if taken on an empty stomach. Corticosteroids tend to suppress the immune system, thereby increasing the risk of infections.

A nurse is caring for a patient who has Addison's disease. The nurse should assess the patient for which symptoms? Select all that apply. 1 Weight gain 2 Hyperpigmentation 3 Weakness and fatigue 4 Orthostatic hypotension 5 Thin skin with ecchymosis

2 Hyperpigmentation 3 Weakness and fatigue 4 Orthostatic hypotension Hyperpigmentation, orthostatic hypotension, and weakness coupled with fatigue are all manifestations of Addison's disease. A patient with Addison's disease will have weight loss, not weight gain. Thin skin with ecchymosis is a manifestation of Cushing syndrome, not Addison's disease.

A patient is diagnosed with adrenocortical insufficiency. What would the nurse anticipate the patient's laboratory findings to look like? Select all that apply. 1 Serum sodium: 140 mEq/L 2 Serum potassium: 6.5 mEq/L 3 Blood glucose levels: 80 mg/dL 4 Blood urea nitrogen (BUN): 30 mg/dL 5 Electrocardiogram (ECG): Peaked T waves

2 Serum potassium: 6.5 mEq/L 3 Blood glucose levels: 80 mg/dL 5 Electrocardiogram (ECG): Peaked T waves Adrenocortical insufficiency leads to hyperkalemia, hypoglycemia, peaked T waves in ECG, hyponatremia, and increased blood urea nitrogen levels. The normal level of serum sodium is 135 to 145 mEq/L, serum potassium is 3.5 to 5 mEq/L, glucose level is 120 to 160 mg/dL, and blood urea nitrogen is 15 to 50 mg/dL. A serum potassium level of 6.5 mEq/L shows increased serum potassium levels (hyperkalemia). A blood glucose level of 80 mg/dL shows decreased glucose levels (hypoglycemia). Peaked T waves are observed in electrocardiogram due to hyperkalemia.

A nurse is caring for a patient admitted for hyperthyroidism. What laboratory results will the nurse expect to see in the electronic chart to confirm hyperthyroidism? Select all that apply. 1 Elevated TSH level 2 Undetectable TSH level 3 Low free thyroxine (free T4) level 4 Elevated free thyroxine (free T4) level 5 Low thyroid-stimulating hormone (TSH) level

2 Undetectable TSH level 4 Elevated free thyroxine (free T4) level 5 Low thyroid-stimulating hormone (TSH) level The primary laboratory findings to confirm the diagnosis of hyperthyroidism are low or undetectable TSH levels and elevated free thyroxine levels. Low free thyroxine levels and elevated TSH levels are found with hypothyroidism.

The nurse is taking a health history for a client with hyperparathyroidism. Which question would elicit information about this client's condition? 1. "Do you have tremors in your hands?" 2. "Are you experiencing pain in your joints?" 3. "Do you notice swelling in your legs at night?" 4. "Have you had problems with diarrhea lately?"

2. "Are you experiencing pain in your joints?"

The nurse has provided instructions to the client with hyperparathyroidism regarding home care measures to manage the symptoms of the disease. Which statement by the client indicates a need for further instruction? 1. "I should avoid bed rest." 2. "I need to avoid doing any exercise at all." 3. "I need to space activity throughout the day." 4. "I should gauge my activity level by my energy level."

2. "I need to avoid doing any exercise at all." Rationale: The client with hyperparathyroidism should pace activities throughout the day and plan for periods of uninterrupted rest. The client should plan for at least 30 minutes of walking each day to support calcium movement into the bones. The client should be instructed to avoid bed rest and use energy levels as a guide to activity. The client also should be instructed to avoid high-impact activity or contact sports.

The emergency department nurse is preparing a plan for initial care of a client with a diagnosis of hyperglycemic hyperosmolar state (HHS). The nurse understands that the hyperglycemia associated with this disorder results from which occurrence? 1. Increased use of glucose 2. Overproduction of insulin 3. Increased production of glucose 4. Increased osmotic movement of water

3. Increased production of glucose Rationale: Hyperglycemia results from decreased use and increased production of glucose. Increased use of glucose and overproduction of insulin would most likely cause hypoglycemia. Option 4 is incorrect.

The nurse has provided home care measures to the client with diabetes mellitus regarding exercise and insulin administration. Which statement by the client indicates a need for further instruction? 1. "I should always wear a Medic-Alert bracelet." 2. "I should perform my exercise at peak insulin time." 3. "I should always carry a quick-acting carbohydrate when I exercise." 4. "I should avoid exercising at times when a hypoglycemic reaction is likely to occur."

2. "I should perform my exercise at peak insulin time." Rationale: The client should be instructed to avoid exercise at peak insulin time because this is when a hypoglycemic reaction is likely to occur. If exercises are performed at this time, the client should be instructed to eat an hour before the exercise and drink a carbohydrate liquid. The remaining options are correct statements regarding exercise, insulin, and diabetic control.

A client with Cushing's syndrome verbalizes concern to the nurse regarding the appearance of the buffalo hump that has developed. Which statement should the nurse make to the client? 1. "Don't be concerned; this problem can be covered with clothing." 2. "Usually these physical changes slowly improve following treatment." 3. "This is permanent, but looks are deceiving and are not that important." 4. "Try not to worry about it; there are other things to be concerned about."

2. "Usually these physical changes slowly improve following treatment." Rationale: The client with Cushing's syndrome should be reassured that most physical changes resolve with treatment. All other options are not therapeutic responses.

The nurse is preparing a plan of care for a client with diabetes mellitus who has hyperglycemia. The nurse places highest priority on which client problem? 1. Lack of knowledge 2. Inadequate fluid volume 3. Compromised family coping 4. Inadequate consumption of nutrients

2. Inadequate fluid volume Rationale: An increased blood glucose level will cause the kidneys to excrete the glucose in the urine. This glucose is accompanied by fluids and electrolytes, causing an osmotic diuresis leading to dehydration. This fluid loss must be replaced when it becomes severe. Options 1, 3, and 4 are not related specifically to the information in the question.

A client with diabetes mellitus is at risk for a serious metabolic disorder from the breakdown of fats for conversion to glucose. The nurse plans care for the client, knowing that pathological fat metabolism is occurring if the client has elevated levels of which substance? 1. Glucose 2. Ketones 3. Glucagon 4. Lactate dehydrogenase

2. Ketones

The nurse is monitoring a client diagnosed with acromegaly who was treated with transsphenoidal hypophysectomy and is recovering in the intensive care unit. Which findings should alert the nurse to the presence of a possible postoperative complication? Select all that apply. 1. Anxiety 2. Leukocytosis 3. Chvostek's sign 4. Urinary output of 800 mL/hour 5. Clear drainage on nasal dripper pad

2. Leukocytosis 4. Urinary output of 800 mL/hour 5. Clear drainage on nasal dripper pad Rationale: Acromegaly results from excess secretion of growth hormone, usually caused by a benign tumor on the anterior pituitary gland. Treatment is surgical removal of the tumor, usually with a sublingual transsphenoidal complete or partial hypophysectomy. The sublingual transsphenoidal approach is often through an incision in the inner upper lip at the gum line. Transsphenoidal surgery is a type of brain surgery and infection is a primary concern. Leukocytosis, or an elevated white count, may indicate infection. Diabetes insipidus is a possible complication of transsphenoidal hypophysectomy. In diabetes insipidus there is decreased secretion of antidiuretic hormone and clients excrete large amounts of dilute urine. Following transsphenoidal surgery, the nasal passages are packed and a dripper pad is secured under the nares. Clear drainage on the dripper pad is suggestive of a cerebrospinal fluid leak. The surgeon should be notified and the drainage should be tested for glucose. A cerebrospinal fluid leak increases the postoperative risk of meningitis. Anxiety is a nonspecific finding that is common to many disorders. Chvostek's sign is a test of nerve hyperexcitability associated with hypocalcemia and is seen as grimacing in response to tapping on the facial nerve. Chvostek's sign has no association with complications of sublingual transsphenoidal hypophysectomy.

The nurse has developed a postoperative plan of care for a client who had a thyroidectomy and documents that the client is at risk for developing an ineffective breathing pattern. Which nursing intervention should the nurse include in the plan of care? 1. Maintain a supine position. 2. Monitor neck circumference every 4 hours. 3. Maintain a pressure dressing on the operative site. 4. Encourage deep breathing exercises and vigorous coughing exercises.

2. Monitor neck circumference every 4 hours. Rationale: After thyroidectomy, neck circumference is monitored every 4 hours to assess for the occurrence of postoperative edema. The client should be placed in an upright position to facilitate air exchange. A pressure dressing is not placed on the operative site because it may restrict breathing. The nurse should monitor the dressing closely and should loosen the dressing if necessary. The nurse should assist the client with deep-breathing exercises, but coughing is minimized to prevent tissue damage and stress to the incision.

The nurse is caring for a client who is 2 days postoperative following an abdominal hysterectomy. The client has a history of diabetes mellitus and has been receiving regular insulin according to capillary blood glucose testing four times a day. A carbohydrate-controlled diet has been prescribed but the client has been complaining of nausea and is not eating. On entering the client's room, the nurse finds the client to be confused and diaphoretic. Which action is most appropriate at this time? 1. Call a code to obtain needed assistance immediately. 2. Obtain a capillary blood glucose level and perform a focused assessment. 3. Ask the unlicensed assistive personnel (UAP) to stay with the client while obtaining 15 to 30 g of a carbohydrate snack for the client to eat. 4. Stay with the client and ask the UAP to call the health care provider (HCP) for a prescription for intravenous 50% dextrose.

2. Obtain a capillary blood glucose level and perform a focused assessment.

The nurse caring for a client with a diagnosis of hypoparathyroidism reviews the laboratory results of blood tests for this client and notes that the calcium level is extremely low. The nurse should expect to note which on assessment of the client? 1. Unresponsive pupils 2. Positive Trousseau's sign 3. Negative Chvostek's sign 4. Hyperactive bowel sounds

2. Positive Trousseau's sign Rationale: Hypoparathyroidism is related to a lack of parathyroid hormone secretion or a decreased effectiveness of parathyroid hormone on target tissues. The end result of this disorder is hypocalcemia. When serum calcium levels are critically low, the client may exhibit Chvostek's and Trousseau's signs, which indicate potential tetany. The remaining options are not related to the presence of hypocalcemia.

Which clinical manifestation is a classic finding in Graves' disease? 1 Gingivitis 2 Cretinism 3 Exophthalmos 4 Muscular dystrophy

3 Exophthalmos Exophthalmos is the protrusion of eyeballs from the orbits; it results from increased fat deposits and fluid in orbital tissues. It is a classic clinical manifestation in Graves' disease. Gingivitis, cretinism, and muscular dystrophy are not classic clinical manifestations associated with Graves' disease.

A patient diagnosed with hyperthyroidism received radioactive iodine one week ago. The patient tells the nurse, "I don't think the medication is working, I don't feel any different." What is the best response by the nurse? 1 "You should notify your primary health care provider immediately." 2 "You may need to have your thyroid removed sooner than anticipated." 3 "It may take several weeks to see the full benefits of the treatment." 4 "You don't feel any different? Would you like to sit down and talk about it?"

3 "It may take several weeks to see the full benefits of the treatment." Radioactive iodine has a delayed response, and the maximum effect may not be seen for up to three months. For this reason, it would not be necessary to contact the primary health care provider immediately, or for the patient to have the thyroid gland removed sooner. Asking the patient to sit and talk about it demonstrates that the nurse is being responsive to psychosocial/emotional needs, but is not the best nursing response at this time.

A patient scheduled for a thyroidectomy is placed on potassium iodine. When the patient's family asks the nurse why this medication is needed, what is the nurse's best response? 1 "This medication will promote thyroid hormone synthesis." 2 "This medication will enhance healing following surgery." 3 "This medication will decrease the vascularity of the thyroid gland." 4 "This medication will inhibit the production of parathyroid hormone."

3 "This medication will decrease the vascularity of the thyroid gland." When a patient is to undergo a thyroidectomy, before surgery antithyroid drugs, iodine, and adrenergic blockers may be administered to achieve a euthyroid state. Iodine reduces vascularization of the thyroid gland, reducing the risk of hemorrhage. Potassium iodide does not promote thyroid hormone synthesis, inhibit the production of parathyroid hormone, or enhance healing and electrolyte balance postoperatively.

The nurse is caring for patients with thyroid cancer. The nurse recognizes that the one with the poorest prognosis is the patient with which type of cancer? 1 Papillary thyroid cancer 2 Follicular thyroid cancer 3 Anaplastic thyroid cancer 4 Medullary thyroid cancer

3 Anaplastic thyroid cancer The patient with anaplastic thyroid cancer has a poor prognosis because the cancer is aggressive and resistant to therapy. Papillary, follicular, and medullary thyroid cancers are treated successfully when compared to anaplastic thyroid cancer.

A patient who underwent thyroid surgery develops neck swelling. What is the first action that the nurse should take? 1 Monitor calcium levels 2 Evaluate difficulty in speaking 3 Assess the patient for signs of hemorrhage 4 Place the patient in a semi-Fowler's position

3 Assess the patient for signs of hemorrhage The patient who undergoes thyroid surgery is at risk for hemorrhage. Swelling is a clinical manifestation of hemorrhage. The first nursing action is to assess the patient. Monitoring calcium levels and evaluating difficulty in speaking helps in assessing the signs of hypoparathyroidism. Placing the patient in a semi-Fowler's position helps in avoiding flexion of the neck and tension on the suture lines.

The nurse is caring for a patient who is postoperative following a thyroidectomy. A priority of the patient's nursing care includes which action? 1 Assessment of hoarseness 2 Assessment of Babinski's reflex 3 Assessment of Chvostek's sign 4 Assessment of neck full range of motion

3 Assessment of Chvostek's sign A positive Chvostek's sign is a sign of life-threatening tetany, which could be caused by hypocalcemia because of accidental removal of the parathyroid glands. Hoarseness for three to four weeks postoperatively is an expected outcome of a thyroidectomy. A Babinski's reflex is not related to thyroid removal. Although it is advisable that the postoperative thyroidectomy patient exercise the neck muscles, neck flexion is contraindicated because it places tension on the suture line.

Which condition shows a clinical presentation of purplish red striae? 1 Hypofunction of androgens 2 Hyperfunction of androgens 3 Hypofunction of glucocorticoids 4 Hyperfunction of glucocorticoids

3 Hypofunction of glucocorticoids Purplish red striae are seen in Cushing syndrome, which occurs due to the hypofunctioning of adrenal cortex. Purplish red striae are observed in a patient with Cushing syndrome due to the hypofunction of glucocorticoids. Hyperfunctioning of androgens may result in hirsutism and hyperpigmentation. Hypofunctioning of androgens may result in decreased axillary and pubic hair in women. Hyperfunctioning of glucocorticoids may result in bronzed skin or hyperpigmentation of face, neck, and hands.

In developing a teaching plan for the patient with exophthalmos, the nurse understands that the highest priority is placed on 1 Avoiding eyestrain 2 Improving self-esteem 3 Preventing corneal injury 4 Minimizing the risk of nerve damage

3 Preventing corneal injury The patient with exophthalmos may not be able to close the eyelids completely. This puts the patient at risk for dry eyes, for overexposure to environmental irritants, and for corneal injury. Lubricating eye drops can be used to combat drying, and dark glasses are encouraged to decrease exposure to environmental irritants. Preventing corneal injury is the priority for the patient with exophthalmos. Exophthalmos may create a function limitation in extraocular movements because of forward protrusion of the globe of the eye. The patient with exophthalmos is encouraged to move the eyes through the six cardinal fields of gaze several times a day to maintain ocular muscle flexibility. Avoiding eyestrain is not a priority for the patient with exophthalmos. Patients may suffer from decreased self-esteem because of the physical changes associated with exophthalmos. Good grooming is encouraged as a strategy to improve self-esteem. Improving self-esteem is of lower priority than preventing corneal injury. Exophthalmos is not associated with ocular nerve damage.

A patient has been taking oral prednisone for the past several weeks after having an exacerbation of asthma. The nurse has explained the procedure for gradual reduction rather than sudden cessation of the drug. What is the rationale for this approach to drug administration? 1 Prevention of hypothyroidism 2 Prevention of diabetes insipidus 3 Prevention of adrenal insufficiency 4 Prevention of cardiovascular complications

3 Prevention of adrenal insufficiency Sudden cessation of corticosteroid therapy can precipitate life-threatening adrenal insufficiency. Diabetes insipidus, hypothyroidism, and cardiovascular complications are not common consequences of suddenly stopping corticosteroid therapy.

The nurse is performing an assessment on a client with pheochromocytoma. Which assessment data would indicate a potential complication associated with this disorder? 1. A urinary output of 50 mL/hour 2. A coagulation time of 5 minutes 3. A heart rate that is 90 beats/minute and irregular 4. A blood urea nitrogen level of 20 mg/dL (7.1 mmol/L)

3. A heart rate that is 90 beats/minute and irregular Rationale: Pheochromocytoma is a catecholamine-producing tumor usually found in the adrenal medulla, but extraadrenal locations include the chest, bladder, abdomen, and brain; it is typically a benign tumor but can be malignant. Excessive amounts of epinephrine and norepinephrine are secreted. The complications associated with pheochromocytoma include hypertensive retinopathy and nephropathy, myocarditis, increased platelet aggregation, and stroke. Death can occur from shock, stroke, kidney failure, dysrhythmias, or dissecting aortic aneurysm. An irregular heart rate indicates the presence of a dysrhythmia. A coagulation time of 5 minutes is normal. A urinary output of 50 mL/hour is an adequate output. A blood urea nitrogen level of 20 mg/dL (7.1 mmol/L) is a normal finding.

The nurse should include which interventions in the plan of care for a client with hyperthyroidism? Select all that apply. 1. Provide a warm environment for the client. 2. Instruct the client to consume a low-fat diet. 3. A thyroid-releasing inhibitor will be prescribed. 4. Encourage the client to consume a well-balanced diet. 5. Instruct the client that thyroid replacement therapy will be needed. 6. Instruct the client that episodes of chest pain are expected to occur.

3. A thyroid-releasing inhibitor will be prescribed. 4. Encourage the client to consume a well-balanced diet. Rationale: The clinical manifestations of hyperthyroidism are the result of increased metabolism caused by high levels of thyroid hormone. Interventions are aimed at reduction of the hormones and measures to support the signs and symptoms related to an increased metabolism. The client often has heat intolerance and requires a cool environment. The nurse encourages the client to consume a well-balanced diet because clients with this condition experience increased appetite. Iodine preparations are used to treat hyperthyroidism. Iodine preparations decrease blood flow through the thyroid gland and reduce the production and release of thyroid hormone. Thyroid replacement is needed for hypothyroidism. The client would notify the health care provider if chest pain occurs because it could be an indication of an excessive medication dose.

The nurse is reviewing the postoperative prescriptions for a client who had a transsphenoidal hypophysectomy. Which health care provider's (HCP's) prescriptions, if noted on the record, would indicate the need for clarification? 1. Assess vital signs and neurological status. 2. Instruct the client to avoid blowing his nose. 3. Apply a loose dressing if any clear drainage is noted. 4. Instruct the client about the need for a MedicAlert bracelet.

3. Apply a loose dressing if any clear drainage is noted. Rationale: The nurse should observe for clear nasal drainage; constant swallowing; and a severe, persistent, generalized, or frontal headache. These signs and symptoms indicate cerebrospinal fluid leak into the sinuses. If clear drainage is noted after this procedure, the HCP needs to be notified. Therefore, clarification is needed regarding application of a loose dressing. The remaining options indicate appropriate postoperative interventions.

The nurse is providing instructions regarding insulin administration for a client newly diagnosed with diabetes mellitus. The health care provider has prescribed a mixture of Humulin N and Humulin R insulin. The nurse should instruct the client that which is thefirst step in this procedure? 1. Draw up the correct dosage of Humulin N insulin into the syringe. 2. Draw up the correct dosage of Humulin R insulin into the syringe. 3. Inject air equal to the amount of Humulin N prescribed into the vial of Humulin N insulin. 4. Inject air equal to the amount of Humulin R prescribed into the vial of Humulin R insulin.

3. Inject air equal to the amount of Humulin N prescribed into the vial of Humulin N insulin.

The nurse is providing instructions regarding insulin administration for a client newly diagnosed with diabetes mellitus. The health care provider has prescribed a mixture of NPH insulin and regular insulin. The nurse should instruct the client that which is the first step in this procedure? 1. Draw up the correct dosage of NPH insulin into the syringe. 2. Draw up the correct dosage of regular insulin into the syringe. 3. Inject air equal to the amount of NPH insulin prescribed into the vial of NPH insulin. 4. Inject air equal to the amount of regular insulin prescribed into the vial of regular insulin.

3. Inject air equal to the amount of NPH insulin prescribed into the vial of NPH insulin. Rationale: The initial step in preparing an injection of insulin that is a mixture of NPH and regular insulin is to inject air into the NPH insulin bottle equal to the amount of insulin prescribed. The client would then be instructed to inject an amount of air equal to the amount of prescribed insulin into the regular insulin bottle. The regular insulin would then be withdrawn, followed by the NPH insulin. Contamination of regular insulin with NPH insulin will convert part of the regular insulin into a longer acting form.

A client is admitted to a hospital with a diagnosis of diabetic ketoacidosis (DKA). The initial blood glucose level was 950 mg/dL. A continuous intravenous infusion of short-acting insulin is initiated, along with intravenous rehydration with normal saline. The serum glucose level is now 240 mg/dL. The nurse would next prepare to administer which item? 1. Ampule of 50% dextrose 2. NPH insulin subcutaneously 3. Intravenous fluids containing dextrose 4. Phenytoin (Dilantin) for the prevention of seizures

3. Intravenous fluids containing dextrose Rationale: Emergency management of DKA focuses on correcting fluid and electrolyte imbalances and normalizing the serum glucose level. If the corrections occur too quickly, serious consequences, including hypoglycemia and cerebral edema, can occur. During management of DKA, when the blood glucose level falls to 250 to 300 mg/dL (14.2 to 17.1 mmol/L), the IV infusion rate is reduced and a dextrose solution is added to maintain a blood glucose level of about 250 mg/dL (14.2 mmol/L), or until the client recovers from ketosis. Fifty percent dextrose is used to treat hypoglycemia. NPH insulin is not used to treat DKA. Phenytoin is not a usual treatment measure for DKA.

A client is brought to the emergency department in an unresponsive state, and a diagnosis of hyperglycemic hyperosmolar state (HHS) is made. The nurse would immediately prepare to initiate which anticipated health care provider's prescription? 1. Endotracheal intubation 2. 100 units of NPH insulin 3. Intravenous infusion of normal saline 4. Intravenous infusion of sodium bicarbonate

3. Intravenous infusion of normal saline Rationale: The primary goal of treatment in hyperosmolar hyperglycemic syndrome (HHS) is to rehydrate the client to restore fluid volume and to correct electrolyte deficiency. Intravenous (IV) fluid replacement is similar to that administered in diabetic ketoacidosis (DKA) and begins with IV infusion of normal saline. Regular insulin, not NPH insulin, would be administered. The use of sodium bicarbonate to correct acidosis is avoided because it can precipitate a further drop in serum potassium levels. Intubation and mechanical ventilation are not required to treat HHS.

A nurse is assisting a client with diabetes mellitus who is recovering from diabetic ketoacidosis (DKA) to develop a plan to prevent a recurrence. Which is most important to include in the plan of care? 1. Test urine for ketone levels. 2. Eat six small meals per day. 3. Monitor blood glucose levels frequently. 4. Receive appropriate follow-up health care.

3. Monitor blood glucose levels frequently.

The nurse is preparing to care for a client after parathyroidectomy. The nurse should plan for which action for this client? 1. Maintain an endotracheal tube for 24 hours. 2. Administer a continuous mist of room air or oxygen. 3. Place in a flat position with the head and neck immobilized. 4. Use only a rectal thermometer for temperature measurement.

3. Place in a flat position with the head and neck immobilized.

The nurse performs a physical assessment on a client with type 2 diabetes mellitus. Findings include a fasting blood glucose level of 120 mg/dL, temperature of 101° F, pulse of 88 beats/minute, respirations of 22 breaths/minute, and blood pressure of 100/72 mm Hg. Which assessment would be of most concern to the nurse? 1. Pulse 2. Respiration 3. Temperature 4. Blood pressure

3. Temperature

The nurse performs a physical assessment on a client with type 2 diabetes mellitus. Findings include a fasting blood glucose level of 120 mg/dL (6.8 mmol/L), temperature of 101°F (38.3°C), pulse of 102 beats/minute, respirations of 22 breaths/minute, and blood pressure of 142/72 mm Hg. Which finding would be the priority concern to the nurse? 1. Pulse 2. Respiration 3. Temperature 4. Blood pressure

3. Temperature Rationale: In the client with type 2 diabetes mellitus, an elevated temperature may indicate infection. Infection is a leading cause of hyperosmolar hyperglycemic syndrome in the client with type 2 diabetes mellitus. The other findings are within normal limits.

The nurse is monitoring a client for signs of hypocalcemia after thyroidectomy. Which sign/symptom, if noted in the client, wouldmost likely indicate the presence of hypocalcemia? 1. Bradycardia 2. Flaccid paralysis 3. Tingling around the mouth 4. Absence of Chvostek's sign

3. Tingling around the mouth Rationale: After thyroidectomy the nurse assesses the client for signs of hypocalcemia and tetany. Early signs include tingling around the mouth and in the fingertips, muscle twitching or spasms, palpitations or arrhythmias, and Chvostek's and Trousseau's signs. Bradycardia, flaccid paralysis, and absence of Chvostek's sign are not signs of hypocalcemia.

The nurse is caring for a client after thyroidectomy. The nurse notes that calcium gluconate is prescribed for the client. The nurse determines that this medication has been prescribed for which purpose? 1. To treat thyroid storm 2. To prevent cardiac irritability 3. To treat hypocalcemic tetany 4. To stimulate release of parathyroid hormone

3. To treat hypocalcemic tetany Rationale: Hypocalcemia, resulting in tetany, can develop after thyroidectomy if the parathyroid glands are accidentally removed during surgery. Manifestations develop 1 to 7 days after surgery. If the client develops numbness and tingling around the mouth, fingertips, or toes; muscle spasms; or twitching, the health care provider is notified immediately. Calcium gluconate should be readily available in the nursing unit.

The nurse has provided dietary instructions to a client with a diagnosis of hypoparathyroidism. The nurse should instruct the client that it is acceptable to include which item in the diet? 1. Fish 2. Cereals 3. Vegetables 4. Meat and poultry

3. Vegetables Rationale: The client with hypoparathyroidism is instructed to follow a calcium-rich diet and to restrict the amount of phosphorus in the diet. Vegetables are allowed in the diet. The client should limit meat, poultry, fish, eggs, cheese, and cereals.

A multidisciplinary health care team is developing a plan of care for a client with hyperparathyroidism. The nurse should include which priority intervention in the plan of care? 1. Restrict fluids to 1000 mL per day. 2. Describe the use of loperamide (Imodium). 3. Walk down the hall for 15 minutes three times a day. 4. Describe the administration of aluminum hydroxide gel.

3. Walk down the hall for 15 minutes three times a day.

A patient with adrenocortical insufficiency develops Addisonian crisis. What should be the immediate nursing action? 1 Administer fludrocortisone daily. 2 Advise an increased intake of salt. 3 Decrease the glucocorticoid dosage. 4 Administer large volumes of saline and dextrose.

4 Administer large volumes of saline and dextrose. Addisonian crisis is a life-threatening emergency in which the patient has low levels of adrenal hormones, leading to a loss of water and sodium. The first course of action is to reverse hypotension by administering large volumes of saline and dextrose. Administration of fludrocortisone can be administered once hypotension is corrected. Increasing the salt in the diet would not have an immediate effect during the Addisonian crisis. Glucocorticoids are given as a long-term therapy.

Which hormone deficiency may lead to a life-threatening condition? 1 Prolactin 2 Oxytocin 3 Follicle-stimulating hormone (FSH) 4 Adrenocorticotropic hormone (ACTH)

4 Adrenocorticotropic hormone (ACTH) Adrenocorticotropic hormone (ACTH) may lead to acute adrenal insufficiency and shock. This may result in a life-threatening situation because of sodium and water depletion. Prolactin plays a role in lactation. Oxytocin is a hormone that is particularly functional during and after childbirth. Follicle-stimulating hormone (FSH) is associated with reproduction and is responsible for the development of eggs in females and sperm in males. The absence of these other hormones are not life threatening.

The nurse should monitor for increases in which laboratory value in a patient being treated with dexamethasone? 1 Sodium 2 Calcium 3 Potassium 4 Blood glucose

4 Blood glucose Hyperglycemia, or increased blood glucose level, is an adverse effect of corticosteroid therapy. Sodium, calcium, and potassium levels are not affected directly by dexamethasone.

The nurse reviews lab values for a patient who underwent thyroidectomy 48 hours ago. Which finding is of most concern? 1 Increased thyroxine 2 Decreased phosphorus 3 Increased serum calcium 4 Decreased serum calcium

4 Decreased serum calcium During thyroid surgery the parathyroid glands are often unavoidably removed. The result is an inability to regulate serum calcium, stemming from a lack of parathyroid hormone. In hypoparathyroidism there is a decrease in parathyroid hormone, which results in decreased serum calcium and increased phosphorus levels. An increase in thyroxine is not seen after thyroidectomy; the thyroxine level may actually drop below normal. Decreased phosphorus and increased serum calcium levels may occur initially after a thyroidectomy because of manipulation of the thyroid gland during surgery. This causes a surge of parathormone, but the level does decrease if the parathyroid glands are removed.

A patient has developed Cushing syndrome due to the prolonged administration of corticosteroid hormonal therapy. What course of action should be taken to treat the patient? 1 Withholding therapy for few days 2 Conversion to an alternate-day regimen 3 Abrupt discontinuance of corticosteroids 4 Gradual discontinuance of corticosteroids

4 Gradual discontinuance of corticosteroids Corticosteroid hormone doses should be decreased gradually until the discontinuation of therapy if the therapy leads to Cushing syndrome. The therapy should not be withheld for a few days. Alternate-day regimen cannot be applied for hormonal therapy. Discontinuing the therapy suddenly might lead to adrenal insufficiency, which is life threatening.

The nurse is caring for a patient with a history of hyperthyroidism who was admitted into the hospital with a kidney infection. It is most important that the nurse notify the health care provider if noting which physical sign or symptom? 1 Flank pain 2 Frequent voiding 3 Heart rate of 100 beats/minute (bpm) 4 Jugular vein distention

4 Jugular vein distention The patient is at risk for thyrotoxicosis because of a medical history of hyperthyroidism and a current diagnosis of infection. Jugular vein distention is a sign of congestive heart failure, which is a manifestation of thyrotoxicosis. Early treatment is essential to prevent further complications. Flank pain and frequent voiding are non-life-threatening manifestations of a kidney infection. A heart rate of 100 bpm needs to be monitored, but is not considered severe tachycardia.

A patient is scheduled for a total thyroidectomy. What information does the nurse include when teaching this patient about recovery after the procedure? 1 Exercise will be restricted for up to six months. 2 A low- or no-sodium diet will be prescribed. 3 Physical therapy will need to be continued. 4 Life-long hormone replacement will be needed.

4 Life-long hormone replacement will be needed. This patient will need life-long thyroid hormone replacement with levothyroxine because the entire thyroid gland will be missing after surgery. Exercise will not be restricted for six months. Lengthy exercise restriction or physical therapy generally is not indicated following a thyroidectomy. A sodium-restricted diet would not ordinarily be necessary.

A patient is scheduled for a bilateral adrenalectomy. What does the nurse include in the discharge teaching for this patient? 1 No replacement therapy will be needed. 2 Weekly adrenocorticotropic hormone (ACTH) injections will be needed. 3 Cortisol will be required if the patient has stress. 4 Lifelong replacement of corticosteroids will be required.

4 Lifelong replacement of corticosteroids will be required. Discharge instructions are based on the patient's lack of endogenous corticosteroids and resulting inability to physiologically react to stressors. Patients undergoing a bilateral adrenalectomy will require lifetime replacement therapy. ACTH injections are not an option, because both adrenal glands were removed during surgery. Exogenous cortisol is required at all times, and the dose needs to be increased dramatically if the patient experiences stress.

In developing a teaching plan for the patient with Addison's disease, what is the nurse's highest priority? 1 Avoiding infection 2 Following a low-salt diet 3 Practicing stress management techniques 4 Managing lifelong corticosteroid replacement

4 Managing lifelong corticosteroid replacement The patient with Addison's disease experiences hypofunctioning of the adrenal cortex, resulting in decreased production of glucocorticoids, mineral corticoids, and androgens. Patients with Addison's disease require lifelong glucocorticoid and mineral corticoid replacement therapy to avoid Addisonian crisis. Addisonian crisis is characterized by profound hypotension, dehydration, fever, tachycardia, hyponatremia, and hyperkalemia. Circulatory collapse may occur if the patient is treated inadequately. Although Addisonian crisis often is triggered by illness-related physiologic stress, and although avoiding infection is important, avoiding infection is of lower priority than managing lifelong corticosteroid replacement. Corticosteroid replacement must be increased during times of stress to prevent Addisonian crisis. Patients taking a mineralocorticoid should increase their salt intake. Emotional stress may contribute to the need for increased corticosteroid replacement. Stress management techniques are important. Practicing stress management techniques, however, is of lower priority than managing lifelong corticosteroid replacement.

The patient with an adrenal hyperplasia is returning from surgery for an adrenalectomy. For what immediate postoperative risk should the nurse plan to monitor the patient? 1 Vomiting 2 Infection 3 Thromboembolism 4 Rapid blood pressure changes

4 Rapid blood pressure changes The risk of hemorrhage is increased with surgery on the adrenal glands as well as large amounts of hormones being released in the circulation, which may produce hypertension and fluid and electrolyte imbalances for the first 24 to 48 hours after surgery. Vomiting, infection, and thromboembolism may occur postoperatively with any surgery.

Which statement is true about pheochromocytoma? 1 The primary treatment is drug therapy. 2 An attack is provoked by antiepileptic medications. 3 Decreased levels of epinephrine and norepinephrine are observed. 4 Severe pounding headaches and profuse sweating are clinical features.

4 Severe pounding headaches and profuse sweating are clinical features. Severe pounding headache and profuse sweating are clinical features of pheochromocytoma. Although drug therapy is administered during preoperative care to reduce complications, the primary treatment is surgery. The attack is provoked by opioids, not antiepileptic medications. Epinephrine and norepinephrine levels rise in patients with pheochromocytoma.

The patient experiencing thyrotoxicosis asks the nurse why he or she is being given propranolol. What is the most accurate answer to the patient's question? 1 To suppress thyroid hormone secretion 2 To prevent thyroid hormone induced hypotension 3 To decrease thyroid gland vascularity in preparation for surgery 4 To block the sympathetic nervous system response to excess thyroid hormone

4 To block the sympathetic nervous system response to excess thyroid hormone Thyrotoxicosis is an acute crisis state of hyperthyroidism often precipitated by a physiologic stressor in the patient with hyperthyroidism. Thyrotoxicosis is an extreme state of hypermetabolism. Excessive amounts of thyroid hormone are present and tissue sensitivity to sympathetic nervous system stimulation is increased, resulting in a number of signs and symptoms, including severe tachycardia leading to heart failure. Propranolol is a beta-adrenergic antagonist that blocks the thyroid-hormone-induced sympathetic nervous system stimulation, resulting in a lowered heart rate and a decreased risk of heart failure. One of the priority treatment goals in the patient with thyrotoxicosis is to decrease thyroid hormone secretion. A decrease in thyroid hormone secretion is primarily accomplished through the use of either methimazole or propylthiourical. Propranolol does not suppress thyroid hormone secretion. In addition to slowing heart rate, propranolol decreases blood pressure; it is not used to prevent hypotension. Nonradioactive strong iodine solution, either in the form of saturated solution of potassium iodine or Lugol's solution, may be used to decrease size and vascularity of the thyroid gland in preparation for surgery. Potassium Iodide or Lugol's solution also may inhibit thyroid hormone synthesis. Propranolol does not decrease size or vascularity of the thyroid gland.

The home care nurse visits a client with a diagnosis of hyperparathyroidism who is taking furosemide (Lasix) and provides dietary instructions to the client. Which statement by the client indicates a need for additional instruction? 1. "I need to eat foods high in potassium." 2. "I need to drink at least 2 to 3 L of fluid daily." 3. "I need to eat small, frequent meals and snacks if nauseated." 4. "I need to increase my intake of dietary items that are high in calcium."

4. "I need to increase my intake of dietary items that are high in calcium." Rationale: The aim of treatment in the client with hyperparathyroidism is to increase the renal excretion of calcium and decrease gastrointestinal absorption and bone resorption of calcium. Dietary restriction of calcium may be used as a component of therapy. The client should eat foods high in potassium, especially if the client is taking furosemide. Drinking 2 to 3 L of fluid daily and eating small, frequent meals and snacks if nauseated are appropriate instructions for the client.

The nurse provides dietary instructions to a client with diabetes mellitus regarding the prescribed diet. Which statement, if made by the client, indicates a need for further teaching? 1. "I'll eat a balanced meal plan." 2. "I need to drink diet soft drinks." 3. "I'll snack on fruit instead of cake." 4. "I need to purchase special dietetic foods."

4. "I need to purchase special dietetic foods."

The nurse has provided instructions for measuring blood glucose levels to a client newly diagnosed with diabetes mellitus who will be taking insulin. The client demonstrates understanding of the instructions by identifying which method as the best method for monitoring blood glucose levels? 1. "I will check my blood glucose level every day at 5:00 pm." 2. "I will check my blood glucose level 1 hour after each meal." 3. "I will check my blood glucose level 2 hours after each meal." 4. "I will check my blood glucose level before each meal and at bedtime."

4. "I will check my blood glucose level before each meal and at bedtime." Rationale: The most effective and accurate measure for testing blood glucose is to test the level before each meal and at bedtime. If possible and feasible, testing should be done during the nighttime hours. Checking the level after the meal will provide an inaccurate assessment of diabetes control. Checking the level once daily will not provide enough data to control the diabetes mellitus.

The nurse provides instructions to a client newly diagnosed with type 1 diabetes mellitus. The nurse recognizes accurate understanding of measures to prevent diabetic ketoacidosis when the client makes which statement? 1. "I will stop taking my insulin if I'm too sick to eat." 2. "I will decrease my insulin dose during times of illness." 3. "I will adjust my insulin dose according to the level of glucose in my urine." 4. "I will notify my health care provider (HCP) if my blood glucose level is higher than 250 mg/dL."

4. "I will notify my health care provider (HCP) if my blood glucose level is higher than 250 mg/dL." Rationale: During illness, the client with type 1 diabetes mellitus is at increased risk of diabetic ketoacidosis, due to hyperglycemia associated with the stress response and due to a typically decreased caloric intake. As part of sick day management, the client with diabetes should monitor blood glucose levels and should notify the HCP if the level is higher than 250 mg/dL (14.2 mmol/L). Insulin should never be stopped. In fact, insulin may need to be increased during times of illness. Doses should not be adjusted without the HCP's advice and are usually adjusted on the basis of blood glucose levels, not urinary glucose readings.

An external insulin pump is prescribed for a client with diabetes mellitus and the client asks the nurse about the functioning of the pump. The nurse bases the response on which information about the pump? 1. Is timed to release programmed doses of short-duration or NPH insulin into the bloodstream at specific intervals 2. Continuously infuses small amounts of NPH insulin into the bloodstream while regularly monitoring blood glucose levels 3. Is surgically attached to the pancreas and infuses regular insulin into the pancreas, which in turn releases the insulin into the bloodstream 4. Gives a small continuous dose of short-duration insulin subcutaneously, and the client can self-administer a bolus with an additional dose from the pump before each meal

4. Gives a small continuous dose of short-duration insulin subcutaneously, and the client can self-administer a bolus with an additional dose from the pump before each meal Rationale: An insulin pump provides a small continuous dose of short-duration (rapid- or short-acting) insulin subcutaneously throughout the day and night. The client can self-administer an additional bolus dose from the pump before each meal as needed. Short-duration insulin is used in an insulin pump. An external pump is not attached surgically to the pancreas.

A client arrives in the hospital emergency department complaining of severe thirst and polyuria. The client tells the nurse that she has a history of diabetes mellitus. A blood glucose level is drawn, and the result is 685 mg/dL. Which intervention should the nurse anticipate to be initially prescribed for the client? 1. Glucagon via the subcutaneous route 2. Glyburide (DiaBeta) via the oral route 3. Insulin aspart via the subcutaneous route 4. Humulin R insulin (regular insulin) via the intravenous (IV) route

4. Humulin R insulin via the intravenous (IV) route Rationale: The client is most likely in diabetic ketoacidosis (DKA). Regular insulin via the IV route is the preferred treatment for DKA. Regular insulin is a short-acting insulin and can be given intravenously; it is titrated to the client's high blood glucose levels. Glucagon is used to treat hypoglycemia, and glyburide is an oral hypoglycemic agent used to treat type 2 diabetes mellitus. Insulin aspart is a short-acting insulin and is not appropriate for the emergency treatment of DKA.

The nurse is reviewing the record of a client admitted to the hospital with a diagnosis of pheochromocytoma. The nurse reads the assessment findings and expects to note documentation of which major symptom associated with this condition? 1. Glycosuria 2. Diaphoresis 3. Weight loss 4. Hypertension

4. Hypertension Rationale: Hypertension is the major symptom associated with pheochromocytoma. Glycosuria, weight loss, and diaphoresis also are clinical manifestations of pheochromocytoma; however, they are not major symptoms.

A 38-year-old patient who has type 1 diabetes plans to swim laps daily at 1:00 PM. The clinic nurse will plan to teach the patient to A. Check glucose level before, during, and after swimming. B. Delay eating the noon meal until after the swimming class. C. Increase the morning dose of neutral protamine Hagedorn (NPH) insulin. D. Time the morning insulin injection so that the peak occurs while swimming.

A - The change in exercise will affect blood glucose, and the patient will need to monitor glucose carefully to determine the need for changes in diet and insulin administration. Because exercise tends to decrease blood glucose, patients are advised to eat before exercising. Increasing the morning NPH or timing the insulin to peak during exercise may lead to hypoglycemia, especially with the increased exercise.

A 55-year-old female patient with type 2 diabetes has a nursing diagnosis of imbalanced nutrition: more than body requirements. Which goal is most important for this patient? A. The patient will reach a glycosylated hemoglobin level of less than 7%. B. The patient will follow a diet and exercise plan that results in weight loss. C. The patient will choose a diet that distributes calories throughout the day. D. The patient will state the reasons for eliminating simple sugars in the diet.

A - The complications of diabetes are related to elevated blood glucose, and the most important patient outcome is the reduction of glucose to near-normal levels. The other outcomes also are appropriate but are not as high in priority.

A patient receives aspart (NovoLog) insulin at 8:00 AM. Which time will it be most important for the nurse to monitor for symptoms of hypoglycemia? A. 10:00 AM B. 12:00 AM C. 2:00 PM D. 4:00 PM

A - The rapid-acting insulins peak in 1 to 3 hours. The patient is not at a high risk for hypoglycemia at the other listed times, although hypoglycemia may occur.

The nurse determines a need for additional instruction when the patient with newly diagnosed type 1 diabetes says which of the following? A. "I can have an occasional alcoholic drink if I include it in my meal plan." B. "I will need a bedtime snack because I take an evening dose of NPH insulin." C. "I can choose any foods, as long as I use enough insulin to cover the calories." D. "I will eat something at meal times to prevent hypoglycemia, even if I am not hungry."

C - Most patients with type 1 diabetes need to plan diet choices very carefully. Patients who are using intensified insulin therapy have considerable flexibility in diet choices but still should restrict dietary intake of items such as fat, protein, and alcohol. The other patient statements are correct and indicate good understanding of the diet instruction.

A 28-year-old male patient with type 1 diabetes reports how he manages his exercise and glucose control. Which behavior indicates that the nurse should implement additional teaching? A. The patient always carries hard candies when engaging in exercise. B. The patient goes for a vigorous walk when his glucose is 200 mg/dL. C. The patient has a peanut butter sandwich before going for a bicycle ride. D. The patient increases daily exercise when ketones are present in the urine.

D - When the patient is ketotic, exercise may result in an increase in blood glucose level. Type 1 diabetic patients should be taught to avoid exercise when ketosis is present. The other statements are correct.


संबंधित स्टडी सेट्स

English Civil War/ Glorious Revolution 1649-1688

View Set

Week 5 assessment - Chapters 33 & 34

View Set

Contemporary Nursing 6th Edition Chapters 8,9,, Townsend: Chapter 18: Anxiety, Obsessive-Compulsive, and Related Disorders (Nursing II), Mental Health Nursing- Medications, Essentials of Psychiatric Mental Health Nursing, 8th Edition Chapter 15, Town...

View Set

section 11 unit 2: Fair Housing and Antitrust Laws in South Carolina

View Set

Unification of Italy and Germany

View Set

Module 1 Chapter 12: Assessment and Care of Patients with Acid-Base Imbalances

View Set

ATI Testing and Remediation Beginning Test

View Set